You are on page 1of 66

"SUGGESTED ANSWERS/HINTS"

Chartered Accountancy Examination


(December 2014)

CAP II Examination

Education Department
The Institute of Chartered Accountants of Nepal
Satdobato, Lalitpur, P.O.Box: 5289, Kathmandu, Nepal
Tel: +977-1-5530832, 5530730, Fax: +977-1-5550774
E-mail: ican@ntc.net.np, Website: www.ican.org.np
The suggested answers/hints published herein do not constitute the basis for evaluation of the
students’ answers in the examination. The answerds are prepared by concerned Resource Persons
and complied by the Education Department of the Institute with a view to assist the students in
their education. While due care is taken in preparation of answers, if any errors or omissions are
noted, the same may be brought to attention of the Education Department. The council of the
Institute is not in any way responsible for the correctness or otherwise of the answers published
herewith.

Publisher : The Institute of Chartered Accountants of Nepal


Satdobato, Lalitpur, P.O.Box: 5289, Kathmandu
Tel: 977-1-5530832, 5530730, Fax: 977-1-5550774
E-mail: ican@ntc.net.np, Website: www.ican.org.np

© The Institute of Chartered Accountants of Nepal

Price : NRs. 40/- (Rupees Fourty only)

Published in April 2015

CONTENT Page No.


Paper 1 Advanced Accounting 1-11
Paper 2 Audit & Assurance 12-18
Paper 3 Corporate & Other Law 19-25
Paper 4 Financial Management 26-35
Paper 5 Cost & Management Accounting 36-46
Paper 6 (A) Business Communication 47-50
Paper 6 (B) Marketing 51-54
Paper 7 Income Tax & VAT 55-64

Printed at:
Print and Art Service
Bagbazar, Kathmandu
Tel: 4244419, 4239154
Suggested Answers - Advanced Accounting CAP-II Examination - December 2014

Advanced Accounting

1. X Ltd. and Y Ltd. amalgamate to form a new company XY Ltd. The Financial Position of these two companies on
the date of amalgamation was as under:
Capital and Liabilities X Ltd.(Rs.) Y Ltd.(Rs.) Assets X Ltd.(Rs.) Y Ltd. (Rs.)
Share Capital Goodwill 80,000
Equity Share of Rs. 100 Land and
each 800,000 300,000 Building 450,000 300,000
7% Preference Share of Plant
Rs. 100 each 400,000 300,000 &Machinery 620,000 500,000
Furniture and
5 % Debenture 200,000 - Fittings 60,000 20,000
General Reserve - 100,000 Sundry Debtors 275,000 175,000
Profit and Loss account 431,375 97,175 Stores and Stock 225,000 140,000
Sundry Creditors 100,000 210,000 Cash at Bank 120,000 55,000
Secured Loan - 200,000 Cash in Hand 41,375 17,175
Preliminary
Expense 60,000
Total 1,931,375 1,207,175 Total 1,931,375 1,207,175

The terms of amalgamation are as under:


a) The assumption of liabilities of both companies.
Issue of 5 preference share of Rs.20 each in XY Ltd. @ Rs. 18 paid up at premium of Rs. 4 per share for each
preference share held.Issue of 6 Equity shares of Rs. 20 each in XY Ltd. @ Rs. 18 paid up at a premium of
Rs. 4 per share for each equity share held in both companies .In addition, necessary cash should be paid to
equity shareholders of both companies as is required to adjust the rights of shareholder of both the
Companies in accordance with intrinsic value of the share of both the companies.
Issue of such amount of fully paid 6 % debenture in XY Ltd. as is sufficient to discharge the 5 % debenture in
X Ltd. at a discount of 5% after takeover.

b) The assets and liabilities are to be taken at book values stock and debtors for which provision at 2 % and
2.5% respectively to be made.
The XY Ltd. to issue 15,000 new equity shares of Rs. 20 each Rs. 18 paid up at a premium of Rs. 4 per share
so as to have sufficient working capital.

Required: 20
Prepare Ledger accounts in the books of X Ltd. and Y Ltd. to close their books.
Answer
Books of X Ltd.
Realization Account
Dr Cr.
Particulars Rs. Particulars Rs.
To Goodwill 80,000.00 By 5% debenture 200,000.00
To land and Building 450,000.00 By Sundry Creditors 100,000.00
To Plant and machinery 620,000.00 By XY Ltd (purchase Consideration) 1,560,000.00
To Furniture and Fitting 60,000.00 By Equity Shareholders A/c( Loss) 51,375.00
To sundry debtors 275,000.00
To stores and stock 225,000.00
To cash at bank 120,000.00
To Cash in Hand 41,375.00
To preference shareholder(excess payment) 40,000.00
1,911,375.00 1,911,375.00

Equity Shareholder account


Dr Cr
Particulars Rs. Particulars Rs.
To preliminary Expenses 60,000.00 By share Capital 800,000.00
To Realization Account 51,375.00 By Profit and Loss A/c 431,375.00
To Equity Share in XY Ltd 1,056,000.00
To Cash 64,000.00
1,231,375.00 1,231,375.00

The Institute of Chartered Accountants of Nepal 1


Suggested Answers - Advanced Accounting CAP-II Examination - December 2014

XY Ltd
Dr. Cr.
Particulars Rs. Particulars Rs.
To Realisation A/C 1,560,000.00 By Equity Shares in xy Ltd
For Equity 10,56,000
For Preference 440000 1,496,000.00
By cash 64,000.00
1,560,000.00 1,560,000.00

Books of Y Ltd
Realization Account
Particulars Rs. Particulars Rs.
To land and Building 300,000.00 By Sundry Creditors 210,000.00
To Plant and machinery 500,000.00 By Secured Loan 200,000.00
To Furniture and Fitting 20,000.00 By XY Ltd ( purchase Consideration) 790,000.00
To sundry debtors 175,000.00 By Equity shareholder A/c (loss) 37,175.00
To stores and stock 140,000.00
To cash at bank 55,000.00
To Cash in Hand 17,175.00
To preference shareholder 30,000.00
1,237,175.00 1,237,175.00

XY Ltd Account
Particulars Rs. Particulars Rs.
To Realisation A/C 790,000.00 By Equity Shares in XY Ltd
For Equity 396000
For Preference 330000 726,000.00
By cash 64,000.00
790,000.00 790,000.00

Equity Shareholder Account


Particulars Rs. Particulars Rs.
To Equity Share in XY Ltd 396,000.00 By Share Capital 300,000.00
To Realisaation 37,175.00 By profit and Loss A/C 97,175.00
To cash 64,000.00 By Reserve 100,000.00
497,175.00 497,175.00

Working Note 1
Purchase consideration X Ltd (Rs.) Y Ltd (Rs.)
Payable to preference shareholder
Preference share @ 22 per share 440,000.00 330,000.00
Equity Shares at Rs 22 per share 1,056,000.00 396,000.00
cash ( WN 2 ) 64,000.00 64,000.00
1,560,000.00 790,000.00

The Institute of Chartered Accountants of Nepal 2


Suggested Answers - Advanced Accounting CAP-II Examination - December 2014

Working Note 2
Value of Net Assets X Ltd Y Ltd
Goodwill 80,000.00
land and Building 450,000.00 300,000.00
Plant and machinery 620,000.00 500,000.00
Furniture and Fittings 60,000.00 20,000.00
Debtor Less 2.5% 268,125.00 170,625.00
Stock less 2 % 220,500.00 137,200.00
Cash at Bank 120,000.00 55,000.00
Cash in Hand 41,375.00 17,175.00
1,860,000.00 1,200,000.00
Less
Debenture 200,000.00 -
Creditors 100,000.00 210,000.00
Secured Loan - 200,000.00
1,560,000.00 790,000.00
Payable in Shares 1,496,000.00 726,000.00
Payable in cash 64,000.00 64,000.00

2.
a) Gold, Silver and Diamond are partners in a firm sharing profits and losses in the ratio of 5:3:2. Gold retires on
32.03.2071 on which date the balance sheet of the firm stood as under:

Liabilities Amount (Rs.) Assets Amount (Rs.)


Capital Accounts: Goodwill 18,000
Gold 190,000 Furniture 45,000
Silver 120,000 Office Equipments 138,000

Diamond 100,000 410,000 Building 202,000


General Reserve 68,000 Stock 176,000
Sundry Creditors 182,000 Sundry Debtors 78,000
Bills Payable 14,000 Cash 17,000

674,000 674,000

It is provided in the deed of the partnership that in the event of the death or retirement of a partner, goodwill is to
be valued at 2 years’ purchase of the Average Profit of the last 4 years. The profits for the last 4 years are:

(Rs.)
• 2070-71: 46,000
• 2069-70: 40,000
• 2068-69: 10,000
• 2067-68: (28,000)

Furniture and Building are revalued at Rs. 40,000 and Rs. 225,000 respectively, stock is overvalued by 10%. It
was decided that Gold should be paid Rs. 140,000 immediately on retirement and balance on his Capital account
is to be treated as a loan to the firm. Silver and Diamond contributed necessary sums in equal proportions for
payment to Gold and to leave Rs. 33,000 cash as working capital.

Assuming that above mentioned arrangements is given effect to show the Revaluation Account, Cash Account
and Capital Account of the partners. 10

The Institute of Chartered Accountants of Nepal 3


Suggested Answers - Advanced Accounting CAP-II Examination - December 2014

b) A Kathmandu based Shop opens a new branch in Biratnagar which trades independently of the head office. The
transactions at the branch for the year ended 31st March 2013 are as under:

(Rs.)
Goods supplied by head office 200,000
Purchases from outsiders:-
Credit 155,500
Cash 30,000 185,500
Sales:
Credit 250,500
Cash 46,000 296,500
Cash received from customers 304,500
Cash paid to creditors 142,500
Expenses paid to creditors 89,500
Furniture purchased by the branch on credit 35,000
Cash received from head office initially 40,000
Remittances to head office 110,000

Prepare the branch final accounts after taking the following into consideration: 10
i) Write off depreciation on furniture at 5% per annum for full year.
ii) A remittance of Rs. 20,000 from the branch to the head office is in transit.
iii) The values its closing stock at Rs. 120,000

Answer a)
In the Books of the Firm
Revaluation A/c
Dr. Cr.
Particulars Amount (Rs.) Particulars Amount (Rs.)
To Furniture A/c 5,000.00 By Building A/c 23,000.00
To Stock A/c 16,000.00
To Partners Capital A/c:
Gold 1,000.00
Silver 600.00
Diamond 400.00 2,000.00
23,000.00 23,000.00

Cash A/c
Cr.
Particulars Amount (Rs.) Particulars Amount (Rs.)
To Balance b/d 17,000.00 By Gold Capital A/c 140,000.00
To Partners Capital A/c:
(Bal Figure)
Silver 78,000.00
Diamond 78,000.00 156,000.00 By Balance c/d 33,000.00
173,000.00 173,000.00

Partners Capital A/c


Dr. Cr.
Diamond Diamond
Particulars Gold (Rs) Silver (Rs.) (Rs.) Particulars Gold (Rs.) Silver (Rs.) (Rs.)
To Bank A/c 140,000.00 - - By Balance b/d 190,000.00 120,000.00 100,000.00
To Gold Loan By General Reserve
A/c 93,000.00 - - A/c 34,000.00 20,400.00 13,600.00
To Goodwill
A/c - 20,400.00 13,600.00 By Goodwill A/c 8,000.00 4,800.00 3,200.00
By Revaluation A/c 1,000.00 600.00 400.00
To Balance c/d - 203,400.00 181,600.00 By Cash A/c 78,000.00 78,000.00

233,000.00 223,800.00 195,200.00 233,000.00 223,800.00 195,200.00

The Institute of Chartered Accountants of Nepal 4


Suggested Answers - Advanced Accounting CAP-II Examination - December 2014

Working Notes:
1) Calculation of Goodwill and its Treatment
2067-68 (28,000.00)
2068-69 10,000.00
2069-70 40,000.00
2070-71 46,000.00
Total 68,000.00
Average Profit of the last 4 years 17,000.00
Goodwill 2 years Average Profit 34,000.00

Goodwill already in the books of accounts 18,000.00


Additional Goodwill 16,000.00

Goodwill Account has already been raised by Rs.18,000 therefore before Gold’s retirement it should be raised by Rs.
16,000(34,000-18,000) in the current profit sharing ratio 5:3:2.
Gold 5/10 X 16,000=8,000
Silver 3/10 X 16,000=4,800
Diamond 2/10 X 16,000=3,200
Total 16,000

Therefore Goodwill is to be written off in the new profit sharing Ratio 3:2
Silver 3/5 X 34,000=20,400
Diamond 2/5 X 16,000 =13,600
Total 34,000

Profit Sharing Ratio


Old New
Gold 5 -
Silver 3 3
Diamond 2 2
Total 10 5

2) Actual Value of Stock


Value of Stock =Rs. 176,000 x 100 = Rs. 160000
110
Therefore it is Overvalued By (176,000-160,000) = Rs. 16,000

b) In Branch Book
Trading and Profit & Loss Account for the year ended on 31st March 2013
Dr. Cr.
Particulars Amount Rs. Particulars Amount Rs.
To Goods supplied by head office 200,000 By Sales:
To Purchases: Credit 250,500
Credit 155,000 Cash 46,000 296,500
Cash 30,000 185,500 By Closing Stock 120,000
To Gross Profit c/d 31,000
Total 416,500 Total 416,500
To Expenses 89,500 BY Gross Profit b/d 31,000
To Depreciation on Furniture 1,750 BY Head Office- transfer of net loss 60,250
Total 91,250 Total 91,250

The Institute of Chartered Accountants of Nepal 5


Suggested Answers - Advanced Accounting CAP-II Examination - December 2014

Balance Sheet as on 31st March 2013


Liabilities Amount Assets Amount Rs
Rs
Sundry Trade Creditors Closing Stock 120,000
(155,500-142,500) 13,000 Cash in Transit 20,000
Creditor for Furniture 35,000 Cash in hand 18,500
Advances from Trade Debtors
(304,500-250,500) 54,000
Head Office Account 56,500
Total 158,500 Total 158,500

Cash Book
Liabilities Amount Assets Amount Rs
Rs
To Head Office (Initial Receipt) 40,000 By Cash Purchases 30,000
To Cash Sales 46,000 By Trade Creditors 142,500
To Trade Debtors 304,500 By Expenses 89,500
By Head Office Remittances 110,000
By Balance c/d 18,500

Total 390,500 Total 390,500

3.
a) An electricity company decided to replace some parts of its plant by an improved plant. The plant to be replaced
was built in 2043 for Rs. 4,200,000. It is estimated that it would cost Rs. 7,800,000 to build a new plant of the
same size and capacity.The cost of the new plant as per the improved design was Rs. 12,600,000 and in addition,
material belonging to the old plant valued at Rs. 456,000 was used in the construction of the new plant. The
balance of the plant was sold for Rs. 360,000.
Compute the amount to be written off to revenue and the amount to be capitalized. Also prepare Plant Account
and Replacement Account. 7

b) Explain ‘Non Performing Assets’. 3


th st
c) A Ltd. prepares accounts on 30 September each year, but on 31 December 2013 fire destroyed the greater part
of its stock .Following information was collected from books of account:

Stock as 1.10 .2013 Rs. 29,700,000


Purchase from 1.10.2013 to 31.12.2013 Rs.75,000,000
Wages from 1.10.2013 to 31.12.2013 Rs.33,000,000
Sales from 1.10.2013 to 31.12.2013 Rs.140,000,000

The rate of gross profit is 33.33% on cost .Stock to the value of Rs.3,000,000 was salvaged .Insurance policy was
for Rs.25,000,000.
Additional Information:
i) Stock in the beginning was calculated at 10% less than cost.
ii) A plant was installed by firm’s own workers.They were paid Rs. 500,000 which was included in the wages.
iii) Purchase includes the purchase of plant of Rs. 5,000,000.
You are required to calculate the claim for the loss of stock. 7
d) Write short note about unexpired risk reserve in Insurance Company 3

The Institute of Chartered Accountants of Nepal 6


Suggested Answers - Advanced Accounting CAP-II Examination - December 2014

Answer
a)
(i) Calculation of Amount Chargeable to Revenue
Rs. Rs.
Estimated Current cost of Replacing old plant 7,800,000.00
Less: Value of replacing old Plant 360,000.00

Value of material belonging to the old plant


used in the construction of new plant 456,000.00 816,000.00
Total 6,984,000.00

(ii) Calculation of Amount to be Capitalized


Rs. Rs.
Cost of Building new plant 12,600,000.00

Add: Value of Material belonging to the old


plant used in the Construction of the new Plant 456,000.00 13,056,000.00
Less: Estimate Current Cost of Replacing old Plant 7,800,000.00
Total 5,256,000.00

(iii)
Plant Account
Rs. Rs.
To Balance b/d 4,200,000.00 By Balance c/d 9,456,000.00

To Cost of Construction 4,800,000.00


To Cost of Old Material 456,000.00
9,456,000.00
9,456,000.00

(iv)Replacement Account

To Bank A/c By Bank A/c 360,000.00


(portion to be written out off By Plant A/c 456,000.00
replacement cost) 7,800,000.00 By Revenue A/c 6,984,000.00
(Bal Fig)
7,800,000.00 7,800,000.00

b) NPA is a classification used by financial institutions that refer to loans that are in jeopardy of default. Once the borrower has
failed to make interest or principle payments for 90 days the loan is considered to be a non-performing asset. Non-performing
assets are problematic for financial institutions since they depend on interest payments for income. Troublesome pressure from
the economy can lead to a sharp increase in non-performing loans and often results in massive write-downs.

The nonperforming asset is not yielding any income to the lender in the form of principal and interest payments.
c)
Computation of claim for loss of stock
Stock on the date of fire (31.12.2013) Rs. 30,500,000
Less salvage stock Rs. 3,000,000
Loss of Stock Rs.27,500,000
Amount of claim = insured value/cost of stock on date of fire *loss of stock
= 25,000,000/30,500,000*27,500,000
= 22,541,000

The Institute of Chartered Accountants of Nepal 7


Suggested Answers - Advanced Accounting CAP-II Examination - December 2014

Working Note
Memorandum Trading Account for period from 1.10.2013 to 31.12.2013
Particulars Rs. Particulars Rs.
To opening stock(29700000*100/90) 33,000,000 By sales 140,000,000
To Purchase 75,000,000 By Closing Stock(balancing figure) 30,500,000
Less cost of plant 5,000,000 70,000,000
To wages 33,000,000 32,500,000
Less wages for plant 500,000
To gross profit (33.33% of cost or 25% of sale) 35,000,000
Total 170,500,000 Total 170,500,000

d) As per Rule 15 of the Insurance Regulation 2049, every insurance Business operating,Non Life Insurance Business shall transfer an
amount not less than fifty percent of the Net Insurance Premium shown in Revenue Account to the ‘Unexpired Risk Reserve
account.Such amount shall be allocated for every category of Insurance the Insures operating eg. an insures operating non life
business and accepting risk for Fire Insurance,Marine Insurance Motor insurance and aviation insurance, then the insurer shall
maintain the Unexpired Risk Reserve for each of the fire,marine, motor and aviation insurance.

Such Unexpired Risk reserve shall be recognized as income in the next year except the Unexpired Risk reserve maintained for
Marine Insurance. In case of marine insurance, unexpired risk reserve maintained for it shall not be recognized as income for at
least three year.
4.
a) The Management of KAS Ltd. appointed a new young sales manger towards the end of Ashadh 2070. The new
manager devised a plan to increase sales and profit by reducing selling price and extending credit period to
customers.

The extracts of the operational result are enlisted here below:


2069-70 2070-71
Particulars (Amount in (Rs.) Lakhs) (Amount in (Rs.) Lakhs)
Sales 1,200 1,800
Gross Profit 150 162
Net Profit 60 63
Equity Share Capital 600 800

Explain whether you believe that the profitability for the year ended 2071 has improved as a result of the new
policies adopted by the new manager. You should support your answer with appropriate ratios. 5

b) Differentiate between Government Accounting and Business Accounting. 5

c) From the following information, calculate the value of an equity share: 5


i) The subscribed share capital of a company consists of 10 lakhs 13% preference shares of Rs. 10 each and
Rs. 20 lakhs equity shares of Rs. 10 each. All the shares are fully paid up.
ii) The average annual profits of the company after providing depreciation but before taxation are Rs.
18,000,000. It is considered necessary to transfer Rs. 3,450,000 to general reserve before declaring any
dividend. Rate of taxation is 30%.
iii) The normal return expected by investors on equity shares from the type of business carried on by the
company is 20%.
iv) Ignore corporate dividend Tax.

Answer
a) Calculation of Ratios:

Ratios Formula 2069-70 2070-71


Gross Profit Margin
=Gross Profit / Sales X 100% 12.50% 9.00%
Net Profit Margin
=Net Profit / Sales X 100% 5.00% 3.50%
Return on Equity
=Net Profit / Equity X 100% 10.00% 7.88%

• With the extracts it could be seen that there has been a rocket increase in the sales which has marked a growth of 50%
• However, the Net Profit has marginally increased from Rs. 60 less to Rs. 63 Lacs in absolute terms.
• The company’s Gross Profit Ratio has been reduced from 12.50% to 9.00%.

The Institute of Chartered Accountants of Nepal 8


Suggested Answers - Advanced Accounting CAP-II Examination - December 2014

• The company’s Net Profit Ratio has been reduced from 5.00% to 3.50%; indicating hard time for share holders in future.
• The company’s Return on Equity has reduced from 10% to 7.88%.
• The new credit policy of the Management may force the company to hold huge stocks as well as the figures of the debtors
could be increased, likely resulting to a higher risk of Bad Debts.

Conclusion
The company should review the new Sales Policy so that profitability in relative terms can also be increased.

b)
1. Meaning
The accounting system maintained by the government offices is known as government accounting.
The accounting system maintained by business organizations is known as business accounting.

2. Objective
Government accounting is maintained by the government offices to know the position of public fund.
Business accounting is maintained by business organizations to know the profit or loss and the financial position of the business.

3. Budget
Government accounting strictly follows the government budgeting system.
Business accounting does not follow the government budgeting system.

4. Basis
Government accounting is prepared on cash basis.
Business accounting is prepared on cash as well as accrual basis.

5. Level of Accounting
Government accounting has the system of central level and operating level accounting.
Business accounting has no provision of central level and operating level accounting.

6. Rules and Provisions:


Government accounting is strictly maintained by following the financial rules and provisions of government.
Business accounting is maintained by following the rules and principles of 'Generally Accepted Accounting Principles'.

7. Information:
Government accounting provides information to the government about the receipts, transfer and deposition of public funds.
Business accounting provides information to the concerned parties about the operating result and financial position of the
business.

c)
Rs.
Average Annual profits before tax 18,000,000
Less: Income Tax @ 30% 5,400,000
12,600,000
Less: Transfer to general reserve 3,450,000
Amount available for dividend 9,150,000
Less: Preference dividend @13% on Rs. 10,000,000 1,300,000
Amount available for equity dividend 7,850,000

Rate of Dividend = 7,850,000/20,000,000 x100 = 39.25%


Normal Rate of Dividend = 20%
Value of an equity share = 39.25 / 20 x 10 = 19.63

5.
a) Prudent prepares financial statement to 31st December each Year, on 1st January 20x0 the entity purchased a non
current asset for Rs 1.6 million that had an anticipated useful life of four Years .This asset qualified for
immediate tax relief of 100% of the cost of that asset.

For the year ended 31st December 20x0; the draft account showed a profit before tax of Rs. 2 million. The
directors anticipate that this level of profit will be maintained for the foreseeable future.

The Institute of Chartered Accountants of Nepal 9


Suggested Answers - Advanced Accounting CAP-II Examination - December 2014

Prudent pays tax at a rate of 30%.Apart from the difference caused by the purchase of no current asset in 20x0,
there are no other difference between accounting profit and taxable profit or the tax base and carrying amount of
net asset. Current year tax for the year is Rs 240,000 and for the years 20x1 to 20x3 are Rs. 720,000 for each
year.

You are required to compute the pre, and post tax profits for the Prudent for each of the four Years ending 31st
December 20x0to 20x3 assuming deferred tax is recognized. 5
b) Discuss the treatment of capitalization of borrowing cost as per NAS 08 5
th
c) R has developed a software program during the year to 30 November 2014.The cost of developing the software
was Rs. 50 million.The software is used by the rest of the group and sold to third parties.Net revenue of Rs. 40
million is expected from sale of the software .Which has quickly become market leader in this field. The software
is expected to generate revenue for four years, after which an upgraded version will be developed

Should R recognize the internally developed software as an Intangible asset? 5

Answer a)
Particulars 20x0 (Rs.) 20x1 (Rs.) 20x2 (Rs.) 20x3 (Rs.)
Carrying Amount 1,200.00 800.00 400.00 -
Tax Base - - - -
Temporary difference 1,200.00 800.00 400.00 -

Closing deferred tax


Liability@30% 360.00 240.00 120.00 -

Opening deferred tax Liability - (360.00) (240.00) (120.00)


Charge/(credit) to income 360.00 (120.00) (120.00) (120.00)

Income statement 20x0 (Rs.) 20x1 (Rs.) 20x2 (Rs.) 20x3 (Rs.)
Profit Before tax 2,000.00 2,000.00 2,000.00 2,000.00
Current Tax (240.00) (720.00) (720.00) (720.00)
Deferred Tax (360.00) 120.00 120.00 120.00
Profit after Tax 1,400.00 1,400.00 1,400.00 1,400.00

b)

Borrowing costs that are directly attributable to the acquisition, construction or production of a qualifying asset shall be capitalized
as part of the cost of asset. Borrowing cost are interest and other cost incurred by an entity in connection with the borrowing of
funds.
A qualifying asset is an asset that necessarily takes a substantial period of time to get ready for its intended use or sale.
Commencement of capitalization
The capitalization of borrowing cost as part of qualifying asset shall commence when:
Expenditures for the asset are being incurred;Borrowing costs are being incurred; and
Activities that are necessary to prepare the asset for its intended use or sale are in progress.
Suspension of capitalization
Capitalization of borrowing cost shall be suspended during extended periods in which active development is interrupted.
Cessation of capitalization
Capitalization of borrowing cost shall cease when substantially all the activities necessary to prepare qualifying asset for its
intended use or sale are complete.

c)

NAS 27 prohibits recognizing Internally Generated Goodwill.NAS 27 also requires internally generated intangibles to be
recognized, provided they meet certain criteria. An entity must demonstrate the technical feasibility of the asset, the ability to
complete and use or sell of the asset, the probable future economic benefits of the asset and the availability of adequate technical,

The Institute of Chartered Accountants of Nepal 10


Suggested Answers - Advanced Accounting CAP-II Examination - December 2014

financial and other resource before it can be recognized. It must also be possible to measure the expenditure attributable the asset
reliably and it should be capable of generating cash inflow in excess of cash outflows.
Because the software is generating external revenue, these criteria appear to be met. But only the cost attributable to the
development of software after the development shall qualify for recognition and the amount so recognized shall be amortized over
useful life of 4 years.

6. Write short notes on: (4×2.5=10)

a) Sweat share
b) Firm underwriting
c) Related party transaction
d) Off balance sheet item
Answer
a) Sweat shares means the equity shares issued by company to its employees or directors at a discount or for a consideration other than
cash for providing technical know-how or making available rights in the nature of property rights or value additions.

Such issue must be authorized by a special resolution passed by the company in a general meeting.
The resolution must specify the category of employees etc, to whom shares are to be issued,number of shares, price, etc.

b) ‘Firm underwriting’ signifies a definite commitment to take up a specified number of shares irrespective of the number of shares
subscribed by the public. In such a case, unless it has been otherwise agreed, the underwriter’s liability is determined without
taking into account the number of shares taken up ‘firm’ by him, i.e. the underwriter is obliged to take up:
1. the number of shares he has applied for ‘firm’; and
2. the number of shares he is obliged to take up on the basis of the underwriting agreement.

c) A business deal or arrangement between two parties who are joined by a special relationship prior to the deal. For example, a
business transaction between a major shareholder and the corporation would be deemed a related-party transaction. As per the
Provision of Nepal Accounting Standards, any such transaction should be properly disclosed in the Financial Statement of the
Organization.

d) An asset or debt that does not appear on a company’s balance sheet. Items that are considered off balance sheet are generally ones in
which the company does not have legal claim or responsibility.

For example, loans issued by a bank are typically kept on the bank's books. If those loans are securitized and sold off as
investments, however, the securitized debt is not kept on the bank's books. One of the most common off-balance sheet items is an
operating lease.

The Institute of Chartered Accountants of Nepal 11


Suggested Answers - Audit & Assurance CAP-II Examination - December 2015

Audit & Assurance

1. As an auditor, give your opinions with reasons on the following cases: (4×5=20)

a) Aashriya& Associates has two Partners holding Certificate of practice. It has 14 Public company clients and other small
clients. The firm is in dilemma as to accept the forthcoming audit request of two Public companies.
b) Smart Pvt. Limited sold a Television worth of Rs. 500,000 (exclusive of all taxes) on 32 Ashadh 2071 which includes Rs.
100,000 for servicing fees of the Television for 5 years from the date of sales. The servicing fee is estimated equal
amount for each guaranteed service years. Smart Pvt. Limited booked entire Rs. 500,000 as revenue for financial year
2070/71.
c) A company purchased a plant for Rs. 20 crores on 1 Shrawan 2070. The company has the policy to charge depreciation
at the rate of 10% on such plants on straight line basis. Due to long dispute between the management and the labour of
the company, the factory was closed from Kartik 1, 2070 to Chaitra end 2070. Management has charged depreciation of
Rs. 1 crore on the said plant to the income statement for the year because the newly purchased plant was not used for 6
months in the year.

d) XYZ is a manufacturing company. There was huge fire in the factory of XYZ on 1 Ashoj 2071 and fixed assets of
written down value Rs. 10 crores was lost out of total fixed assets of Rs. 20 crores of the company. The financial
statements of the company for the year 2070/71 was approved by the Board on 30 Ashoj 2071 in which fixed assets have
been presented at WDV of Rs. 20 crores despite the severe loss due to fire and the information about the loss due to fire
is properly explained in the Notes to the financial statements.

Answer

a) The Directive issued by ICAN on Ceiling over the number of audit requires that a member holding COP can audit the books of
accounts of a maximum100 clients only, in a financial year. Out of these 100 clients, number ofPublic Companies shall not exceed
15.The above limit is applicable for each member of a partnership firm.Provided, organizations whose annual turnover is less than
NRs. 2lakhs, such as small Cooperatives, Religious organizations, SocialOrganizations, Consumer Group, Different Committees,
Trade Unions, Professional Associations and other entities of similar nature are notincluded while calculating the above limit.Thus,
in view of above requirement, new audit can be accommodated subject to total limit for two partners.

b) As per NAS 7 "Revenue" when the selling price of a product includes an identifiable amount for subsequent servicing (for
example, after sales support and product enhancement on the sale of software), that amount is deferred and recognized as revenue
over the period during which the service is performed. The amount deferred is that which will cover the expected costs of the
services under the agreement, together with a reasonable profit on those services.Accordingly in the light of aforesaid provision of
NAS 7; the accounting treatment made by Smart Pvt. Limited is not correct. Smart should book Rs. 400,000 as revenue for FY
2070/71 and Rs. 100,000 should be deferred. For each coming year Rs. 20,000 should be recognized as revenue to match its
services cost.

c) Depreciation of an asset begins when it is available for use, i.e. when it is in the location and condition necessary for it to be
capable of operating in the manner intended by management. Depreciation of an asset ceases at the earlier of the date that the asset
is classified as held for sale (or included in a disposal group that is classified as held for sale) in accordance with NAS 06 and the
date that the asset is derecognized. Therefore, depreciation does not cease when the asset becomes idle or is retired from active use
unless the asset is fully depreciated. So, in the given case, depreciation expenses to be charged to the income statement for the year
2070/71 should be Rs 2 crores (i.e. for full year) instead of Rs 1 crore.

d) As per NAS-05, Event After the Balance Sheet Date, events after the reporting period are those events, favorable and unfavorable,
thatoccur between the end of the reporting period and the date when the financial statements are authorized for issue. Two types of
events can be identified:

i. those that provide evidence of conditions that existed at the end of the reporting period (adjusting events after the
reporting period); and
ii. those that are indicative of conditions that arose after the reporting period (non-adjusting events after the reporting period)

So, the event in the given case (fire after reporting period) is a non-adjusting event.An entity shall not adjust the amounts
recognized in its financial statements to reflect non-adjusting events after the reporting period.

If events after the balance sheet date impacts going concern status of the entity, the entity is required to prepare its financial
statement on break-up value basis. This does not seem to be the case here.

The Institute of Chartered Accountants of Nepal 12


Suggested Answers - Audit & Assurance CAP-II Examination - December 2015

If non-adjusting events after the reporting period are material, non-disclosure could influence the economic decisions that users
make on the basis of the financial statements. Accordingly, an entity shall disclose the following for each material category of non-
adjusting event after the reporting period:

i. the nature of the event; and


ii. an estimate of its financial effect, or a statement that such an estimate cannot be made.

So, presenting fixed assets at Rs 20crores in the balance sheet with appropriate disclosure in the Notes to Accounts seems to be
appropriate.

2. Answer the following: (3×5=15)

a) Explain ‘An appropriate subject matter’ as an element of assurance.


b) Explain ‘Audit Sampling’ and ‘Sampling Risk’ in the light of NSA - 530 Audit Sampling and Other Selective Testing
Procedures.
c) Based on study and evaluation of internal control system, auditor concludes that internal controls are well designed
and functioning effectively. Still auditor performs appropriate substantive audit procedures to form an opinion on the
financial statements before issuance of audit report. Explain why auditor has to perform appropriate substantive audit
procedures in such an ideal situation also.

Answer

a) Assurance engagement is an engagement in which a practitioner expresses a conclusion designed to enhance the degree of
confidence of the intended users other than the responsible party about the outcome of the evaluation or measurement of a subject
matter against criteria.

The subject matter, and subject matter information, of an assurance engagement can take many forms, such as:

• Financial performance or conditions (for example, historical or prospective financial position, financial performance and
cash flows) for which the subject matter information may be the recognition, measurement, presentation and disclosure
represented in financial statements.
• Non-financial performance or conditions (for example, performance of an entity) for which the subject matter information
may be key indicators of efficiency and effectiveness.
• Physical characteristics (for example, capacity of a facility) for which the subject matter information may be a
specifications document.
• Systems and processes (for example, an entity’s internal control or IT system) for which the subject matter information may
be an assertion about effectiveness.
• Behavior (for example, corporate governance, compliance with regulation, human resource practices) for which the subject
matter information may be a statement of compliance or a statement of effectiveness.

Subject matters have different characteristics, including the degree to which information about them is qualitative versus
quantitative, subjective versus objective, historical versus prospective, and relates to a point in time or covers a period. Such
characteristics affect the:

(a) Precision with which the subject matter can be evaluated or measured against criteria; and
(b) The persuasiveness of available evidence.
(c)
The assurance report notes characteristics of particular relevance to the intended users.
An appropriate subject matter is:

(a) Identifiable, and capable of consistent evaluation or measurement against the identified criteria; and
(b) Such that the information about it can be subjected to procedures for gathering sufficient appropriate evidence to
support a reasonable assurance or limited assurance conclusion, as appropriate.

b) Audit sampling involves the application of audit procedures to less than 100% of items within an account balance or class of
transactions such that all sampling units have a chance of selection. This will enable the auditor to obtain and evaluate audit
evidence about some characteristic of the items selected in order to form or assist in forming a conclusion concerning the
population from which the sample is drawn. Audit sampling can use either a statistical or a non-statistical approach.

The Institute of Chartered Accountants of Nepal 13


Suggested Answers - Audit & Assurance CAP-II Examination - December 2015

Sampling risk arises from the possibility that the auditor's conclusion, based on a sample may be different from the conclusion
reached if the entire population were subjected to the same audit procedure. There are two types of sampling risk:

i. the risk the auditor will conclude, in the case of a test of control, that control risk is lower than it actually is, or in the
case of a substantive test, that a material error does not exist when in fact it does. This type of risk affects audit
effectiveness and is more likely to lead to an inappropriate audit opinion; and
ii. the risk the auditor will conclude, in the case of a test of control, that control risk is higher than it actually is, or in the
case of a substantive test, that a material error exists when in fact it does not. This type of risk affects audit efficiency
as it would usually lead to additional work to establish that initial conclusions were incorrect.

c) An internal control system can provide only reasonable assurance that the management’s objectives in establishing the system are
achieved. This is due to the fact that internal control system has the following inherent limitations:

i) Controls have to be cost effective. So, some controls may not be instituted because they are not cost effective.
ii) Most controls are directed at transactions of usual or routine nature. Therefore transactions of unusual nature may
escape from ambit of internal control.
iii) The potential of human error remains in any system of control.
iv) Controls may not prevent frauds through collusion between two or more persons.
v) Management may override controls.
vi) Controls may not keep pace with changing circumstances.
vii) Management itself may manipulate transactions or accounting estimates.

Above inherent limitations of internal control system makes it necessary for the auditor to perform substantive procedures to
express an appropriate opinion on the financial statements. But if the "Test of Control" conducted by the auditor concludes that
internal control system is in existence, operated throughout the period and functioning effectively, the auditor may not consider
necessary to perform substantive audit procedure for that particular area. Substantive procedures though cannot be fully eliminated
and the auditor determines the area where substantive procedures should be used.

3. Give your comments on the following: (3×5=15)

a) Chitale Limited has declared dividend of 12% for FY 2069/70 in the month of Ashoj 2070. The company has booked
this as Interim Dividend in Financial Statement of 2069/70.
b) “Responsibility for properly determining the quantity and value of inventories rests with the management of the
entity”.
c) Ramu& Co. had conducted the audit of XYZ Limited for the financial years 2067/68, 2068/69 and 2069/70. CA. Raman
Saha, partner of Ramu& Co, left the firm on 1 Baishakh 2069. CA. Raman Saha has been appointed as the auditor of
XYZ limited for the year 2070/71.

Answer

a) Dividend has been declared at year end so this should be final dividend.

Further, Nepal Accounting Standards 05 on Events after the Balance Sheet Date provides that if an entity declares dividends to
holders of equity instruments after the balance sheet date, the entity shall not recognise those dividends as a liability at the balance
sheet date.

If dividends are declared after the balance sheet date but before the financial statements are authorised for issue, the dividends are
not recognised as a liability at the balance sheet date because they do not meet the criteria of a present obligation. Such dividends
are disclosed in the notes in accordance with NAS 01 Presentation of Financial Statements.

Thus, the treatment followed by Chitale limited is not in line with NAS.

b) As per NSA 200; "Objectives and General Principles Governing an Audit of Financial Statements" the responsibility for preparing
and presenting the financial statements is that of the management of the entity. Accordingly the responsibility for properly
determining the quantity and value of inventories rests with the management of theentity. Therefore, it is the responsibility of the
management of the entity to ensure that inventories included in the financial information are physically in existence and
representall owned by the entity.

The management can satisfy this responsibility by carrying out appropriate procedures such as verification of all items of inventory
at least once in every financial year. The auditor is expected to examine the adequacy of the methods and procedures of physical

The Institute of Chartered Accountants of Nepal 14


Suggested Answers - Audit & Assurance CAP-II Examination - December 2015

verification followed by the entity. He is also required to determine whether the procedures for identifying defective, damaged,
obsolete, excess and slow-moving items are well-designed and operate properly.

This responsibility of the management is not reduced even where the auditor attends any physical count of inventories in order to
obtain audit evidence. The entities usually maintain detailed stock records in the form of Stores/Stock ledgers showing in respect
of each major item the receipts, issues and balances. The extent of examination of these records by an auditor with reference to the
relevant basic documents (e.g., goods received notes, inspection reports, material issue notes, bin cards, etc.) depends upon the
facts and circumstances of each case. In valuation aspects, compliance with NAS 4 should also be ensured.

c) As per section 111(3) of the Companies Act, no auditor or his/her partner or ex-partner or employee or ex-employee shall be
appointed as auditor for more than three consecutive terms to perform the audit of a public company. Provided, however, that this
restriction shall not apply to any partner who ended partnership or any employee who left the service of such auditor three years
before.

In the present case, Ramu& Co has been auditor of XYZ Ltd for three consecutive years till 2069/70. Since Raman Saha is the ex-
partner of Ramu& Co and 3 years has not elapsed from the date of ending his partnership with Ramu& Co (as date of separation
being 1 Baisakh 2069), his appointment as auditor of the company for the year 2070/71 cannot be considered as lawful.

4. Answer the following: (3×5=15)


a) What are the principal contents of Terms of Audit Engagement as per NSA 210?
b) ABC Hydropower Pvt. Ltd. has purchased equipment worth of Rs. 4 million which is kept stand by for urgent usage on
need basis for repairing the heavy equipment as and when default is reported in functioning of heavy equipment. The
accountant has treated it as recurring inventory item and charged to profit and loss account at the end of each financial
year based on consumption pattern calculated on reasonable basis. Is the accounting treatment made by accountant is
correct? Comment.
c) Write down any five audit steps for the audit of Debtors.

Answer

a) The form and content of audit engagement letters may vary for each client, but they would generally include reference to:

• The objective of the audit of financial statements.


• Management's responsibility for the financial statements.
• The scope of the audit, including reference to applicable legislation, regulations or pronouncements of the Institute of
Chartered Accountants of Nepal (ICAN).
• The form of any reports or other communication of results of the engagement
• The fact that because of the test nature and other inherent limitations of an audit, together with the inherent limitations of any
accounting and internal control system, there is an unavoidable risk that even some material misstatement may remain
undiscovered.
• Unrestricted access to whatever records, documentation and other information requested in connection with the audit.
• Management's responsibility for establishing and maintaining effective internal control.

Other matters that may be included are:


• Description of the basis for fees.
• Ownership and accessibility of the auditor’s files to external parties.
• Interactions with specialists, internal auditors, and the predecessor auditor needed to conduct the audit
• Restrictions to the auditor's liability.

b) As per NAS 6- Property, Plant and Equipment, spare parts and servicing equipment are usually carried as inventory and recognized
in profit or loss as consumed. However, major spare parts and stand-by equipment qualify as property, plant and equipment when
an entity expects to use them during more than one period. Similarly, if the spare parts and servicing equipment can be used only in
connection with an item of property, plant and equipment, they are accounted for as property, plant and equipment.

In view of the above provision made in NAS 6, the equipment purchased by ABC Hydropower Pvt. Ltd. should be treated as
property, plant and equipment. Though it is servicing equipment used on repairing heavy equipment; it has to be kept stand-by and
can be used only in connection with heavy equipment and usable for more than one accounting period, it should be treated as
property, plant and equipment instead of treating it as inventory.

The Institute of Chartered Accountants of Nepal 15


Suggested Answers - Audit & Assurance CAP-II Examination - December 2015

c) Five audit steps for the audit of Debtors out of various possible steps are presented below:

i. Verify the opening balances with last year closing audited balances
ii. Obtain a schedule of receivables and determine whether the total agrees withthe trial balance.
iii. Obtain and consider explanations of significant variations in Debtor’s turnover ratio of current period and prior period and
anticipated debtor turnover ratio.
iv. Select a sample of debtors and obtain direct confirmation from the parties concerned.
v. Obtain an aged analysis of the debtors. Inquire about the reason for unusually large accounts, credit balances on accounts
or any other unusual balances and inquire about the collectibility of receivables.

5. Comment on the following situations/statements: (3×5=15)


a) Explain the types of threat as explained in ICAN Code of ethics.
b) "The rendering of two or more types of professional services concurrently does not by itself impair integrity, objectivity
or independence". Comment.

c) Lalu and Associates has been appointed as auditor for the year 2070/71 of ABC Bank Limited, a commercial bank in
the general meeting dated 25 Paush 2070 for remuneration of Rs. 500,000. In the course of the audit, the auditor finds
that the audit fee is too low and negotiates with the management for increase of the fees to the tune of Rs. 700,000. The
board approves the increased fee.

Answer

a) The various types of threats explained in ICAN code of ethics are as follows:

ƒ Self-interest threat – the threat that a financial or other interest willinappropriately influence the professional accountant’s
judgment orbehavior; For e.g. lowballing, hospitality or other benefits, contingent fees, loans to clients etc.

ƒ Self-review threat – the threat that a professional accountant will notappropriately evaluate the results of a previous
judgment made or serviceperformed by the professional accountant, or by another individualwithin the professional
accountant’s firm or employing organization,on which the accountant will rely when forming a judgment as part ofproviding
a current service; For e.g. valuation services along with audit service, accounting services.

ƒ Advocacy threat – the threat that a professional accountant will promotea client’s or employer’s position to the point that the
professionalaccountant’s objectivity is compromised; For e.g. the auditor Should not offer legal services to a client and
defendthem in dispute or litigation which is material to the financial statement.

ƒ Familiarity threat - the threat that due to a long or close relationshipwith a client or employer, a professional accountant will
be toosympathetic to their interests or too accepting of their work. For e.g. participation in client affairs, family and personal
relationship, audit partners leaving to join clients etc.

ƒ Intimidation threat – the threat that a professional accountant willbe deterred from acting objectively because of actual or
perceivedpressures, including attempts to exercise undue influence over theprofessional accountant. For e.g. if there is actual
or threatened litigationbetween client and assurance firm, the firmshould not continue to act

b) Section 11 of Code of Ethics of the Institute of the Chartered Accountants of Nepaldeals with "Activities Incompatible with the
Practice of Public Accountancy". Section 11 (2) stated that "the rendering of two or more types of professional services
concurrently does not by itself impair integrity, objectivity or independence".

As per Section 11 (1) a professional accountant in public practice should not concurrently engage in any business, occupation or
activity which impairs or might impair integrity, objectivity or independence, or the good reputation of the profession and therefore
would be incompatible with the rendering of professional services.

Further as per Section 11 (3) the simultaneous engagement in another business, occupation or activity unrelated to professional
services which have the effect of not allowing the professional accountant in public practice properly to conduct a professional
practice in accordance with the fundamental ethical principles of the accountancy profession should be regarded as inconsistent
with the practice of public accountancy.

c) Asper section 67 of Bank andFinancial Institutions Act, 2063, the remuneration of the auditor shall be as prescribed by the general
meeting if he or she has been appointed by the general meeting and by the board if he or she has been appointed by the board. In

The Institute of Chartered Accountants of Nepal 16


Suggested Answers - Audit & Assurance CAP-II Examination - December 2015

the given case Lalu and Associates has been appointed by the general meeting of ABC Bank Limited and hence his remuneration
shall be as prescribed by general meeting. So, the revision in the audit fees by the board in this case is not valid.

6. Write short notes on the following: (4×2.5=10)


a) Revaluation of Fixed Assets
b) Objectives of Audit of the Financial Statement
c) Ownership and Custody of Working Papers
d) Qualities of an Auditor

Answer

a) Revaluation of fixed assets is the process of increasing or decreasing their carrying value in case of major changes in fair market
value of the fixed asset. NAS-06, Property, Plant and Equipment requires fixed assets to be initially recorded at cost but they allow
two models for subsequent accounting for fixed assets, namely the cost model and the revaluation model. Under the Revaluation
model, the asset is carried at a revalued amount, being its fair value at the date of revaluation less subsequent depreciation and
impairment, provided that fair value can be measured reliably. Revaluations should be carried out regularly, so that the carrying
amount of an asset does not differ materially from its fair value at the balance sheet date. If an item is revalued, the entire class of
assets to which that asset belongs should be revalued. Revalued assets are depreciated in the same way as under the cost model. If a
revaluation results in an increase in value, it should be credited to other comprehensive income and accumulated in equity under
the heading "revaluation surplus" unless it represents the reversal of a revaluation decrease of the same asset previously recognised
as an expense, in which case it should be recognised in profit or loss. A decrease arising as a result of a revaluation should be
recognised as an expense to the extent that it exceeds any amount previously credited to the revaluation surplus relating to the same
asset. When a revalued asset is disposed of, any revaluation surplus may be transferred directly to retained earnings, or it may be
left in equity under the heading revaluation surplus. The transfer to retained earnings should not be made through profit or loss.

b) In conducting an audit of financial statements, the overall objectives of the auditor are:
• To obtain reasonable assurance about whether the financial statements as a whole are free from material misstatement,
whether due to fraud or error, thereby enabling the auditor to express an opinion on whether the financial statements are
prepared, in all material respects, in accordance with an applicable financial reporting framework;and
• To reporton the financial statements,and communicate as required by the NSAs,in accordance with the auditor’s findings.

Audits are not conducted with the objective of identifying fraud and errors prevalent in the company, but the audit process
normally result in identification of fraud or errors while conducting internal control evaluation and using substantive procedures.
This is because the auditing standards requires an auditor to bear in mind the possibility of the existence of frauds or errors in the
accounts under audit since they may cause the financial position to be mis-stated. So, detection of material frauds and errors as an
incidental objective of independent financial auditing flows from the main objective of determining whether or not the financial
statements give a true and fair view.

c) NSA 230, Audit Documentation contains the set of standards that deal with working papers. The standard provides that the auditor
should prepare, on a timely basis, audit documentation that provides a sufficient appropriate record of the basis for the auditor’s
report, andevidence that the audit was performed in accordance with NSAs and applicable legal and regulatory requirements.
Working papers provide evidence that an effective, efficient, and economic audit has been carried out. As such, working papers are
the property of the auditor. The auditor may, at his discretion, make portions of or extracts from his working papers available to his
client. The auditor should retain the working papers for the period specific or the period necessary to fulfill professional
responsibility relating to the assignment.

d) An auditor should adhere to the fundamental principles applicable for the auditor. The fundamental principles are integrity,
objectivity, professional competence and due care, confidentiality and professional behavior. The qualities required in and auditor
are tact, caution, firmness, good temper, integrity, discretion, industry judgment, patience, clear headedness and reliability. In
addition, he must have the shine of culture for attaining a great height. He must have the highest degree of integrity backed by
adequate independence. He must have a thorough knowledge of general principles of law which govern matters with which he is
likely to be intimate contact. He must pursue an intensive program of theoretical education in subjects like financial and
management accounting, general management, business and corporate laws, computer and information systems, taxation,
economics etc. An auditor must be honest; i.e; he must not certify what he does not believe to be true and must take reasonable
care and skill before he believes that what he certifies is true.

The Institute of Chartered Accountants of Nepal 17


Suggested Answers - Audit & Assurance CAP-II Examination - December 2015

7. Distinguish between: (2×5=10)


a) Auditing and Investigation
b) Batch Processing and On-Line Real Time System

Answer

a) Objects: The object of auditing is to find out whether Financial Statements givea true and fair view of business. Investigation is
undertaken to know the essential facts about a matter under inquiry. It is done with some special purpose of view.
Period: Auditing usually covers one accounting year . Investigation may cover more than one accounting year.
Legally Binding :Auditing is conducted for proprietors or is normally legally binding. Investigation is carried out on behalf of any
party interested in the matter.
Scope :Auditing is legally compulsory for companies & restricted to the financial statement. Investigation is voluntaryand carried
out on certain circumstances & may be carried out beyond the financial statement.
Time :Auditing may be conducted at the year end. Investigationmay be conducted atany time in case ofsuspicion about
anytransaction.
Report :In auditing form of report is prescribed. Ininvestigationit is not prescribed.
Appointment : In auditing appointment is made by shareholders in AGM. Ininvestigationappointment can be done by other
delegated authorities.
Qualification :Auditors qualification is prescribed by Laws. Investigators qualification has not prescribed by Laws.
Re-work :Re-audit is not generally carried out . Re-investigation may be carried out.
Perception :Audit is not carried out with doubtful mind . Investigation may be carried out with doubtful mind.

b) Data processing system in computerized environment is generally Batch Processing or On-Line Real Time (OLRT) system which
can be distinguished as under:

Batch Processing On-Line Real Time (OLRT) system


♦ Transactions are accumulated and processed in ♦ Transactions are processed as and when they occur.
group.
♦ Two types of files are maintained master file is ♦ Only master file is maintained. It keeps updating.
updated when batch processing is run.
♦ Updating does not take place as quickly as in On- ♦ Though updating takes place immediately the
Line Real time system. processing becomes complex.
♦ Not useful when instant and updated results are ♦ Useful for immediate reporting system.
required.
♦ Generally provides Audit trail. ♦ Generally, does not provide audit trail and hence
requires more attention of auditor.

The Institute of Chartered Accountants of Nepal 18


Suggested Answers - Corporate & Other Laws CAP-II Examination - December 2014

Corporate & Other Laws

1. Answer the following questions:

a) Miss Sita Sharma, a chartered accountant licensed by ICAN has been appointed as the auditor of XYZ Trading Ltd. for the
fiscal year 2071/72. It has been revealed that she was appointed by the general meeting of thecompany of the fiscal year
2070/71 on the basis of recommendation of her maternal uncle's son who has been working in the company as a general
manager since the last several years. Later, the appointment was questioned as the close relatives of an employee cannot be
appointed as an auditor in the company pursuant to the Companies Act, 2006. Discuss referring to the relevant provisions of the
Companies Act,2063 the validity of her appointment with disqualification of the auditor alongwith its consequence. 10

b) Auditor of Company ABC found out some irregularities during the course of audit and would like the shareholders to know
about these irregularities, therefore, he wants to have the company’s Extra Ordinary General Meeting (EGM) convened. But he
doesn't know how it would be convened,so he approaches you. Could you explain how theEGM of Company ABC can be
convened pursuant tothe Companies Act, 2063? 5

c) PQR Company is making profit for the last two years, the Board of Directors (BOD) of PQR Company wants to issue premium
shares. The BOD consults you regarding the pre-conditions of and the procedures for issuing premium shares. Suggest the
BOD of PQR Company about the pre-conditions to and the procedures for the issue of premium shares under the Companies
Act, 2063, and also subject to it whether PQR Company can premium shares or not. 5

d) A Proposed Sagarmatha silk public limited company applied to the Company Registrar’sOffice (CRO) for the registration of
apublic company with authorized capital Rs. 50 lacks only. After scrutinizing all the relevant papers Company Registrar refused
to register the company. Answer, what was the reason to refuse registration of that proposed company. 5
Answer:
a) Under Section 111(1) of the Companies Act, 2063, an auditor of a company shall be appointed from amongst the auditors
licensed to carry out audit under the prevailing law by the general meeting, in the case of a public company. The auditor shall
hold office until the next annual general meeting. For such appointment the provisions of Memorandum of Association and
Articles of Association of the Company shall also have to be confirmed. 1

Section 112 of the Companies Act, 2063 specifies the disqualifications of auditor as follows: 1) none of the following persons
or the firms or companies in which such persons are partners shall be qualified for appointment as auditor and shall, despite
appointment as auditor, continue to hold office: (1+3=4)
i) A director, advisor appointed with entitlement to regular remuneration or cash benefit, a person or employee or worker
involved in the management of the company or a partner of any of them or employee of any of such partners or a close
relatives of a director or partner, out of them, or an employee of such relative;
ii) A debtor who has borrowed moneys from the company in any manner, or a person who has failed to pay any dues
payable to the company within the time limit and is in such arrears or a close relative of such person;
iii) A person who has been sentenced to punishment for an offense pertaining to audit and a period of five years has not
elapsed thereafter;
iv) A person who has been declared insolvent;
v) A substantial shareholder of the company or a shareholder holding one percent or more of the paid up capital of the
company or his/her close relative;
vi) A person who has been sentenced to punishment for an offense of corruption , fraud or a criminal offense involving
moral turpitude and a period of five years has not elapsed thereafter;
vii) An auditor or his partner or ex- partner or employee or ex- employee who has already served as auditor for more than
three consecutive terms;
viii) In the case of a public company, any person who works, whether full time or part time, for any Governmental body or
anybody owned fully or partly by the Government of Nepal or any other company or a partner of such person or a
person who is working as an employee of such partner or a person who is authorized to sign any documents or reports to
be prepared by the management of the company;
ix) A company or corporate body with limited liability;
x) A person having interest in any transaction with the company or his/her close relative or a director, officer or substantial
shareholder of another company having any interest in any transaction with the company.

Section 2(z9) of Companies Act, 2063 defines the term “close relative” which means a partition shareholder in a joint family
or husband, wife, father, mother, mother-in-law, father-in-law, elder brother, younger brother, elder sister, younger sister,

The Institute of Chartered Accountants of Nepal 19


Suggested Answers - Corporate & Other Laws CAP-II Examination - December 2014

sister-in -law , (elder or younger brother's wife), brother in –law, sister in –law, brother in law(husband of elder sister), uncle,
aunt, maternal uncle, maternal aunt, son, daughter, daughter-in –law, grand-son, grand-daughter, grand-daughter-in law or
son-in-law.

On the basis of the above legal provisions the of the Companies Act, 2063 maternal uncle's son does not include under the
definition of close relative, therefore, Miss Sita Sharma is not disqualified to be appointed as director of XYZ Trading Ltd.,
though she was recommended for appointment by her maternal uncle’s son who was working as General Manager in that
Company.

Section 112(2) of the Companies Act, 2063, states that the auditor shall, prior to her appointment give information in writing
to the company that pursuant to Section 112(3) of this Act she is not disqualified.So Miss Sita Sharma’s prior information to
the company is also okay.
Section 82 of the Companies Act, 2063 has included provisions relating to a Company’s EGM,which are as follows:
Pursuant to section 82(1) of the Companies Act, 2063, the board of directors of a company may convene an extra-ordinary
general meeting (EGM) if it deems necessary.

b) Pursuant to section 82(2) of this Act, if in the course of examining the account of the Company ABC, it is deemed necessary
to call an extra ordinary general meeting for discussion of the irregularities at the EGM, the auditor may request to the board
of directors to call such meeting; and if the board of directors fails to call the meeting accordingly, the auditor may make an
application, setting out the matter, to the Office of Company Registrar (Office) and if an application is so made, the Office
may call the extra-ordinary general meeting of the company.

c) Section 2(z2) of the Companies Act, 2063 defines the term “premium share”as a share issued by a company as to sell it for a
value in excess of its face value.

d) Section 29 of the Companies Act, 2063 providesthe pre conditionsof and procedures to issue premium shares, which are as
follows:
(1) any company fulfilling the following conditions may, with the prior approval of the Company Registrar’s Office
(Office), issue shares at a premium:
a) the company has been making profits and distributing dividends for three consecutive years,
b) the company's net worth exceeds its total liabilities,
c) the company's general meeting has decided to issue shares at a premium

As PQR Company has been making profit for the last two years only, it doesn't meet the pre-condition mentioned above under
section 29(1)(a) of the Companies Act, 2063.Therefore, PQR Company cannot issue premium shares.

Refusal of registrar to register the public limited company is valid in pursuance to Section 11(1) of the Companies Act, 2063.
Pursuant to this section,the paid up capital of a public company shall be a minimum of ten million rupees, except as otherwise
provided in the prevailing law or in a notification by the Government of Nepal in the Nepal Gazette that the paid up capital of
any particular company shall be in excess of the said required minimum.

The Company Registrar’s Office(CRO), pursuant to section 6(1)(d) of the Companies Act, 2063 may refuse to register a
company if the requirements for the incorporation of a company under this Act are not fulfilled. So the CRO refused to register
this proposed public Company for not fulfilling the requirement of section 11(1) of the above Act.

2. Answer the following questions:


A) Nepal Rastra Bank (NRB) takes XYZ Bank under its control after suspending its board of directors as it failed to honour
liability and also it has not been properly operated. It was later on found out that XYZ Bank was able to honour its commitment
and was also in a position to be operated properly. Answer, what action may be taken by NRB under Nepal Rastra Bank Act,
2058 in this situation. 5
B) Mr. A, ex- governor of Nepal Rastra Bank, has been appointed as Director of XYZ Bank Ltd. immediately after his retirement
from the post of governor. His appointment was questioned by one of the shareholders. In this circumstance, whether his
appointment was against the legal provision or not? Answer it on the basis of the Banks and Financial Institutions Act (BAFIA),
2063. 5
C) Dwell upon the criminal liability of a drawer of a cheque in case it committed a crime under the provisions of Nepalese
Negotiable Instruments Act, 2034. 5
Answer:
a) Pursuant to section 86(1) of Nepal Rastra Bank Act,2058, NRB had taken such commercial bank under its control after
suspending its Board of Directors for having that commercial bank failed to honour its liability and also that has not been
properly operated.

The Institute of Chartered Accountants of Nepal 20


Suggested Answers - Corporate & Other Laws CAP-II Examination - December 2014

b) In pursuance to section 86(2) of this Act, NRB may after taking the management of XYZCommercial Bank under its control,
operate its business by itself or appoint a person, firm, company or institution to operate or manage the business of such
commercial bank. NRB shall, within one year of operating the business on its own or through any other person, firm, company
or institution audit or cause to audit the accounts of the concerned commercial bank and publish the report thereof pursuant to
section 86(3)) of this Act.
c) Pursuant to section 86(5) of this Act, NRB may take the following actions if it is convinced from the auditing report that the
concerned commercial bank is able to honour its commitment or such commercial bank is in a position to be operated properly:
i) To release the suspension of the BOD of the commercial bank and to order the same BOD to operate the business; or
ii) To dismiss the BOD of the commercial bank suspended and to have the business conducted by a new BOD formed from
amongst the shareholders of the commercial bank.
iii) To get the new BOD elected after convening the general meeting of the shareholders of that commercial bank and to have
the business conducted by it; or
iv) To take any other appropriate action as NRB may deem fit.

Pursuant to sub-sections (6) and (7) of Section 86 of Nepal Rastra Bank Act, 2058, NRB shall, before taking the commercial
bank under its control, provide an opportunity to defend within fifteen days to such commercial bank as per the situation. The
commercial bank in that situation shall bear all the expenses incurred while operating the commercial bank after NRB has taken
over pursuant to this Section.

d) Yes, Mr. A's appointment as a director of XYZ Bank Ltd. is against the existing legal provision of BAFIA, 2063. Section 18(j)
of the BAFIA, 2063 mentions that governor, deputy governor or special class officer of Nepal Rastra Bank are disqualified to
become Director unless one year of time from the date of such relief or retirement from the service has not elapsed. As Mr. A
has been appointed right after his retirement, one year time span has not been elapsed from his retirement from the post of
governor, he is automatically disqualified to become director of any other commercial bank. Therefore, his appointment as a
director of commercial bank is against section 18(j) of BAFIA2063.

e) Section 2(h) of Negotiable Instruments Act, 2034 defines a “Cheque” as a Bill of Exchange drawn on a certain Bank payable on
demand. Section 107A of the Negotiable instruments Act,2034 imposes the following criminal liability or punishment against a
drawer who commits the following corresponding offence against the Act:
In case any person who deliberately transfers a Cheque by drawing it to somebody that he/she does not bear deposit in the Bank
or even if there is a deposit which is not sufficient, and if the Cheque thus transferred is dishonoured due to lack of sufficient
deposit when the Cheque is presented to the concerned Bank for the payment, the amount mentioned in the Cheque as well as
interest on it shall be caused to be recovered to the Holder from the Drawer and he/she shall be punished with an imprisonment
up to Three months or a fine up toThree Thousand Rupees or both.

3. Answer the following questions:


a) Mr. Z, one of the directors of PQR ltd., passes some secret information regarding the securities of this company to his relatives.
On the basis of that secret information his relatives bought huge number of shares of that company. Few months later, share
price went up and Mr. Z's relatives earned huge amount of money from the transaction of those shares. Did Mr. Z commit any
crime under the Securities Act, 2063? If yes, what crime did he commit? and what punishment could be imposed on him?
Answer it on the basis of Securities Act,2063. 5
b) What are the liabilities of the concerned persons for the matter referred to in the prospectus as per the Securities Act, 2063?
5
Answer:
a) Yes, Mr. Z one of the directors of PQR Ltd. committed a crime against the Securities Act,2063. His activities are known as
insider trading and it is a punishable crime under Nepalese law. Section 91 of the Securities Act,2063 states that 'in case any
person conducts or causes others to conduct transactions in securities on the basis of any inside information or report which
has not been made public and which can have an impact on the value of the securities, or supplies any such information or
report possessed by him to any other person, except in the course of discharging his duties, he shall be deemed to have carried
out an insider trading of securities’.
b) Regarding the punishment, pursuant to section 101 of this Act,Mr. Z a director of PQR Ltd. Company is an insider of this
Companywho deals in securities in contravention of section 91 of this Act, is proved to have done so, he shall be punished
with a fine equal to the amount involved in the offence or with imprisonment for a term not exceeding one year or with both.
1. The liabilities of the concerned personsfor the matters referred to in the prospectus are laid down under Section 33 of the
Securities Act,2063, which are as follows:
2. The concerned body corporate and the director signing a prospectus and the expert preparing such a prospectus shall be
personally and collectively liable for the truth of the details and documents underpinning the information set down in the
prospectus submitted to Nepal Securities Board (Board) for the purpose of registering securities with the Board and
obtaining permission to issue such securities.

The Institute of Chartered Accountants of Nepal 21


Suggested Answers - Corporate & Other Laws CAP-II Examination - December 2014

3. Where any person who subscribes for any securities on the faith of the matters set down in the prospectus subsequently
sustains any loss or damage by the reason that the matters set down in the prospectus have been set down with mala fide
intention or untrue or false statements have been included therein knowingly, the body, director or experts preparing the
prospectus shall be liable to pay compensation for such loss or damage. Provided that no director shall be liable to pay such
compensation if he or she proves that he or she has resigned prior to making a decision on the matters set down in the
prospectus with ulterior motive or knowingly or that he or she did not know that the prospectus was untrue.

4. Where any investor sustains any loss or damage by the reason that the prospectus, information, statements or returns submitted by a
body corporate to the Board, such an investor may make a petition to the concerned District Court for compensation within thirty
five days from the date of knowledge within one year after the making of investment.

5. Answer the following questions:


a) Mr. JamesSaw, a foreign investor invested in Nepal in a joint venture with a Nepali investor and operated an industry since
2012. Later, a dispute arose regarding the sharing of profit and loss between him, Nepali investor and the industry. He wants to
file a case against the Nepali investor and the industry. So he consults you regarding the way of resolving the dispute between
him, the Nepali investor and the industry. Advise him the way of resolving the dispute between him and the industry under the
Foreign Investment and Technology Transfer Act, 2049. 5
b) Enumerate the functions, duties and powers of the Industrial Promotion Board as provided by the Industrial Enterprises Act,
2049. 5
Answer:
a) Section 7 of Foreign Investment and Technology Transfer Act,1992 has provided the way out for solving the dispute between
the foreign investor and the concerned industry. According to it, if any dispute arises between a foreign investor, national
investoror the concerned industry, the concerned parties shall be required to settle the dispute by mutual consultations in the
presence of Department of Industry. If the dispute could not be settled through mutual consultation, it shall be settled by
arbitration in accordance with the prevailing Arbitration Rules of the UNCITRAL. Moreover, the place of arbitration shall be
held in Kathmandu and the Nepalese laws shall be applicable in the arbitration. Therefore, Mr. James Saw should try to settle
the dispute by following the provisions mentioned above, but pursuant to the foreign investment agreement concluded between
him and the national (Nepali) investor.

Notwithstanding anything contained in sub-sections(1),(2) and (3) above, disputes arising in regards to foreign investment
made in the industries with investments as prescribed may be settled as mentioned in the foreign investment agreement.

b) Section 13 of the Industrial Enterprises Act,2049 provides the following power, duties and functions to the Industrial
Promotion Board (Board)
a. To render necessary cooperation in formulating and implementing policies, laws and regulations pertain to the
industrialization of the country.
b. To give guidelines in attaining the objectives of liberal, open and competitive economic policies pursued by the country
so as to make the industrial sector competitive.
c. To maintain coordination between the policy level and the implementation level of the industrial policy.
d. To cause to follow the ways and means for the prevention of the environmental pollution by putting more emphasis on the
avoidance of effects on the environment and the public health.
e. To make recommendation to Government of Nepal for the inclusion of any industry in the classification of industries.
f. To make recommendation to Government of Nepal to introduce changes in the Areas mentioned in Annex-3 by making
evaluation thereof from time to time.
g. To give directives to the concerned body after making enquiries into the application submitted by any industry
complaining that the industry has not received the facilities and concessions to be made available by the committee.
h. Other functions, duties and power of the Board shall be as prescribed.

6. Answer the following questions:


a) Explain the provisions relating to ceiling on bonus payment to be made to the employees as per the Bonus Act,2030. 5
b) Mention the objectives of the Institute of Chartered Accountants of Nepal (ICAN) under Nepal Chartered Accountants Act,
2053. 5
c) What do you understand by the term Corporate Body Substantially owned by the Government of Nepal? Who will audit the
account of such Corporate Body Substantially owned by the Government of Nepal? and how it will be audited? Answer it in the
context of the Audit Act, 2048. (2+3=5)

The Institute of Chartered Accountants of Nepal 22


Suggested Answers - Corporate & Other Laws CAP-II Examination - December 2014

Answer:
a) Ceiling on the distribution of bonus has been mentioned in Section 7(3) of Bonus Act 2030, which are as follows:
b) The management shall assess the percentage of bonus amount to be obtained by on Employee in a Fiscal year.While
computing bonus amountto be received by an employee, the amount so separated for the distribution of bonus by the
enterprise for the particular fiscal year shall be multiplied by a sum of one hundred and the amount so deducted shall be
divided by the pay or wage amount of the employee so entitled to receive the bonus and the amount so deducted shall be the
percentage of bonus amount.
Whatever mentioned in this Act, the bonus to be obtained by an employee shall not exceed the following amounts:
a. An amount equivalent to the salary or wage of Six months, to an employee, who obtains upto Five Thousand Rupees as
salary or wage.
b. An amount equivalent to the salary or wage of Four months to an employee, who obtains Five thousand One Rupees to
Fifteen Thousand Rupees as salary or wage.
c. An amount equivalent to the wage or salary of Three months to an employee who obtains more than Fifteen Thousand
rupees as salary or wage.
d. The minimum bonus amount to be obtained under Clauses (b) and (c) of Sub-section (3)of this section above shall not be
less than the maximum bonus amount to be obtained under Clauses (a) and (b) respectively as above.
i) Section 5 of Nepal Chartered Accountants Act,2053 states the objectives of the ICAN, which are as follows:
ii) To play the role of regulatory body to encourage the members to carry on accounting profession being within the
extent of the code of conduct in order to consolidate and develop accounting profession as a cause for economic
development of the nation.
iii) To enhance social integration and faith in accounting profession by raising awareness of the general public towards
the importance of accounting profession and the economic and social responsibility of professional accountants.
iv) To develop, protect and promote the accounting profession by enabling professional accountants understand their
responsibility towards the importance of accounting profession and accountancy.
v) To develop mechanism of registration, evaluation and examination of accounting professionals in consonance with
international norms and practices so as to make the accounting profession respectable and reliable.

c) Section 2(e) of the Audit Act,2048 defines the term 'corporate body substantially owned by theGovernment of Nepal.
According to this provision corporate body substantially owned by Nepal government means a corporate body whose more
than 50 percent shares or assets are owned by the Government ofNepal. So far as the audit of the account of such body is
concerned, it is mentioned under section 7 of this Act. According to which, the audit of such body shall be done in
accordance with the existing laws relating to such body. For that purpose,the Auditor General shall be consulted while
appointing an auditor for auditing of such body. A copy of the report of audit submitted by such Auditor shall be delivered to
the office of theAuditor Generalby such body. The procedures to be followed while consulting the Auditor General for
appointing auditors and on matters to principles of audit to be followed by the auditors doing their audit shall be as prescribed
by the Auditor General. Pursuant to section 7(5) of this Act, the Auditor General may issue directives to the concerned
organization is respect of the irregularities observed in the report received and it shall be the duty of the concerned
organization to abide by such directives.

7. Answer the following questions:


a) What are the circumstances that makes contract voidable? Answer it in the light of the Contract Act, 2056. 5
b) Explain about the payment of insurance claims after the cancellation of the insurer in the light of Insurance Act, 2049. 5
c) Explain the provisions relating to 'specific performance' of a contract under the Contract Act,2056. 5
Answer:
a) A contract means an agreement enforceable by law concluded between two or more parties for performing or not performing
any work. Voidable contracts are such which are defective in the process of making it, but not really void due to defects in the
essence of it. Voidable contracts can be made enforceable by the aggrieved party, following the prescribed procedures, and
within the prescribed period.

Section 14 of the Contract Act, 2056 mentions few circumstances that makes an agreement voidable, which are as follows:
1. Contracts concluded through coercion: Explanation to section 14(1)(a) of this Act explains it as follows: A person shall be
deemed to have indulged in coercion if he/she, withthe objective of compelling any person, to accept any contract against
his/her will, withholds or threatens to withhold property belonging to him/her, or threatens to defame him/her, or takesor
threatens to take any other action in contravention of prevailing law.

2. Contract concluded through Undue influence: Explanation to section 14(1)(b) of this Act explains it as follows: (1) Undue
influence means influence exercised by a person upon another person who is under his/her influence and is amenable to
his/her personal benefit or interest. (2) without prejudice to the generality of clause (1), the following persons shall be
deemed to be under the influence of any person and amenable to his/her wishes-(i) A person living under his/her

The Institute of Chartered Accountants of Nepal 23


Suggested Answers - Corporate & Other Laws CAP-II Examination - December 2014

guardianship, protection or custody; (ii) Persons who cannot take care of their interest temporarily or permanently by
reason of old age, sickness or physical or mental weakness; (iii) A person who can subjected to under one’s economic or
ranking influences.

3. Contract concluded through fraud: Explanation to section 14(1)(c) of this Act explains it as follows : A party to a contract
or his/her agent shall be deemed to have committed fraudif he/she, leads the other party or his/her agent to believe or takes
any action to believe the particular matter is true, although he/she knows that it is false, orsuppressesany information in
his/her possession, or indulges in any other fraudulentactpunishable under prevailing law, with the intention of deceiving
the opposite party or his/her agent.

4. Contract concluded through deceit: Explanation to section 14(1)(d) of this Act explains it as follows: Any of the following
act shall be taken as deceit:
i. Submission of false particulars on any matter without reasonable basis for doing so;
ii. Misleading any party so as to aggrieve him/her:
iii. Causing any wrong deliberately on the matter of contract.

b) Section 16 of the Insurance Act, 2052provides the following about the payment of insurance claims after the cancellation of the
insurer: After the cancellation of the insurer, the insurer, dissolved by the cause of the cancellation of its registration pursuant to
section 13, shall refund the amount received by it for insurance to the person, organization or theInsuranceBoard (Board), within the
period and method specified by the Board. It shall refund the principal amount alongwith bonus as specified by the Board in the
case of life insurance, and it shall refund the principal amount as specified by the Board on a proportional basisin the case of non-
life insurance.

c) Section 86 of the Contract Act, 2056 provides for ‘Specific Performance’. Accordingly in case the cash compensation paid in
consideration of the actual loss or damage suffered by the aggrieved party as a result of breach of contract is not reasonable or
adequate, the aggrieved party may demand the execution of the contract as stipulated specific performance instead of making a
claim for compensation (section 86(1))
Notwithstanding anything contained in Sub-section (1) above, no claims for execution of the contract as stipulated specific
performance shall be heard in any of the following circumstances;
i. In case the amount paid in cash as compensation for breach of contract is adequate;
ii. In case the court cannot supervise whether or not the work to be performed under the contract has been actually performed;
iii. In case the contract has been signed for providing services relating to personal expertise, skill or knowledge;
iv. In case the situation is -such that the contract cannot be executed as stipulated;
v. In case the party violating the contract him/herself demands that the contract be executed as stipulated.

8. Write short notes on the following: (2x5=10)


a) Powers of Labour Officer
b) Exchange Equalization Fund

Answer:
a) Pursuant to section 64 of the Labour Act, 2048, the Government of Nepal by publishing a notice in the Nepal Gazette may
appoint one or more Labour Officers, as per necessity or designate any other officer to perform the functions of a Labour
Officer for one region/sector.
b) Pursuant to Section 65 of theLabour Act, 2048, the Labour Officer shall have the following powers –
1. To enter into the premises of the Enterprise as per necessity;
2. To examine the documents and registers of the Enterprise relating to workers and employees;
3. To function or advise as per necessity for improving labour relations;
4. To attempt for solving disputes arising between workers or employees and the Proprietor;
5. To implement welfare provisions, if it does not exit and where if exist, to supervise whether or not it is operated properly;
6. To supervise the implementation of minimum remuneration prescribed by Government of Nepal;
7. To record statement, of anybody to fulfill the objectives of this Act, as per necessity;
8. To perform tasks of the Factory Inspector in his/her absence, except technical tasks; and
9. To perform other tasks as per the directives of Government of Nepal and Department of Labour.
10. Other powers, functions and duties shall be as prescribed.

c) Section 45 of the Banks and Financial Institutions Act, 2063 has provision for the Exchange Equalization Fund. Accordingly:
(1) a licensed institution which has obtained the license to carry on foreign exchange transactions shall make necessary
accounts adjustments in the profit and loss account of the revaluation profits earned as a result of fluctuations in the exchange
rates of foreign currencies, other than the Indian currency, every year at the end of the same fiscal year. While making such

The Institute of Chartered Accountants of Nepal 24


Suggested Answers - Corporate & Other Laws CAP-II Examination - December 2014

accounts adjustment in the profit and loss account, if revaluation earning has been made in any fiscal year, at least twenty five
per cent of such profits shall be credited to the exchange equalization fund.

d) Provided that in the case of revaluation profit-loss resulting from fluctuation in the exchange rate of the Indian currency, it
shall be as prescribed by the Rastra Bank.

e) (2) No amount credited to the exchange equalization fund pursuant to Sub-section (1) above shall, without the approval of the
Rastra Bank, be spent or transferred for any purpose other than the adjustment of loss resulting from the devaluation of foreign
currencies.

The Institute of Chartered Accountants of Nepal 25


Suggested Answers - Financial Management CAP-II Examination - December 2014

Financial Management
1. Beta Company Limited is considering replacement of its existing machine by a new machine which is expected to cost Rs. 264,000.
The new machine will have a life of five years and will yield annual cash revenues of Rs. 568,750 and incur annual cash expenses of
Rs. 295,750. The estimated salvage value of the new machine is Rs. 18,200. The existing machine has a book value of Rs. 91,000
and can be sold for Rs. 45,500 today.

The existing machine has a remaining useful life of five years. The annual cash revenues from this machine will be Rs. 455,000 and
associated annual cash expenses will be Rs. 318,500. The existing machine will have a salvage value of Rs. 4,550 at the end of its
useful life.

The company is in 25% tax bracket, and writes off depreciation at 25% on written-down value method.

The company has a target debt to total capital ratio of 15%. It has raised debt at 11% in the past and it can raise fresh debt at 10.5%.
The company plans to follow dividend discount model to estimate the cost of equity capital. It further plans to pay a dividend of Rs.
2 per share in the next year. The dividend per equity share of the company is expected to grow at 8% p.a. The current market price
of the company's equity share is Rs. 20 per equity share.

Required: (4+8+3+4+1=20)
a) Compute the weighted average cost of the capital of the company.
b) Compute the incremental cash flows for replacement decision.
c) Find out the net present value of the replacement decision.
d) Estimate the discounted payback period of the replacement decision.
e) Should the company replace the existing machine? Advise.

Answer
a) Computation of Weightedaverage cost of capital of the company (WACC)
Ke = D1/P0 + g
=2/20 + 0.08
= 0.18 = 18%
Kdt = 10.5%×(1-0.25)
= 7.875%

WACC= Kd×D/(D+E) + Ke×E/(D+E)


=(7.875%×15%) + (18%× 85%)
=1.18% + 15.3%
=16.48%

(b& c) Incremental cash flow and NPV of replacement decision


Particulars Year 0 Year 1 Year 2 Year 3 Year 4 Year 5
Incremental Initial Cash Outlay (WN 1) (207,125)
Incremental Revenue (WN 2) 113,750 113,750 113,750 113,750 113,750
Saving in Expenses (W.N 2) 22,750 22,750 22,750 22,750 22,750
Less: Incremental Depreciation (W.N 3) 43,250 32,437 24,328 18,246 13,685
Earnings before Tax 93,250 104,063 112,172 118,254 122,815
Less: Tax @ 25% 23,313 26,016 28,043 29,564 30,704
Earnings after Tax 69,937 78,074 84,129 88,690 92,111
Add: Incremental Depreciation 43,250 32,437 24,328 18,246 13,685
Add: Incremental salvage value (18200-
4550) 13,650
Add: Incremental Tax Saving on Loss
on Sale(W.N 4) 6,851
Incremental cash flows (207,125) 113,187 110,511 108,457 106,936 126,297
PV Factor@16.48% 1.00 0.860 0.740 0.636 0.547 0.470
Present Value (207,125) 97,341 81,778 68,979 58,494 59,360
NPV 158,827

The Institute of Chartered Accountants of Nepal 26


Suggested Answers - Financial Management CAP-II Examination - December 2014

d) Calculation of Discounted PBP of replacement decision:

Year Cumulative PV (Rs.)


0 (158,827)
1 (109,784)
2 (28,006)
3 40,973
4 99,467
5 158,827

Discounted Payback Period


= 2 Years + Cumulative PV of Cash flows in 2nd Year
PV of Incremental Cash flow in 3rd Year
= 2 years + 28,006/68,979
= 2.406 years or 2 Years 5 months (approx)

e) The company should replace the machine since the incremental NPV of the decision is positive and discounted PBP is much lower
than the life of the machine.

Working Notes:
1. Incremental initial cash outlay

Purchase price of new machine = Rs. 264,000


Less: Current sales price of old machine = Rs. 45,500
BSV- 91,000
CSV- 45,500
Loss 45,500
Tax Saving on due to loss (45,500*25%) 11,375
Incremental initial cash outlay = Rs. 207,125

2. Calculation of incremental revenue and saving in expenses

Annual Incremental revenue = Rs. 568,750 – Rs. 455,000


= Rs. 113,750
Annual Saving in expenses = Rs. 318,500 - Rs. 295,750
=Rs.22,750

3. Calculation of incremental depreciation


Year Depreciation of new machine (Rs.) Depreciation of Old machine (Rs.) Difference
(Rs.)
1 264,000 × 25% = 66,000 91,000×25% = 22,750 43,250
2 198,000× 25%= 49,500 68,250 ×25%=17063 32,437
3 148,500 × 25%= 37,125 51,187 × 25%=127,97 24,328
4 111,375 × 25%= 27,844 38,390 × 25%=9,598 18,246
5 83,531 × 25%= 20,883 28,792 × 25%= 7,198 13,685

4. Incremental loss on sale at the end of 5th year


BV of new machine = Rs. 83,531 –Rs. 20,883 = Rs. 62,648
Less : BV of old machine = Rs. 28,792 –Rs.7,198 = Rs. 21,594
Incremental book value = Rs. 41,054
Less: Incremental sales value = Rs. 18,200 –Rs. 4550 = Rs. 13,650
Incremental loss = Rs. 27,404
Tax savings @ 25% = Rs. 6,851

2.
a) Explain the important ratios that would be used in each of the following situations: 6
i) A bank is approached by a company for a loan of Rs. 50 lakhs for working capital purpose.
ii) A long term creditor interested in determining whether his claim is adequately secured.
iii) A shareholder who is examining his portfolio to decide whether he should hold or sell his holdings in a company.
b) A company has 8 lakhs equity shares outstanding at the beginning of the year. The current market price per share is Rs. 120.
The board of directors of the company is contemplating to declare a dividend of Rs. 6.40 per share. The rate of capitalization,
appropriate to the risk-class to which the company belongs, is 9.6 percent.

The Institute of Chartered Accountants of Nepal 27


Suggested Answers - Financial Management CAP-II Examination - December 2014

Required: (4+5=9)
i) Based on M-M approach, calculate the market price of the share of the company, when dividend is (i) declared and (ii) not
declared.
ii) How many new shares are to be issued by the company, if the company desires to fund an investment budget of Rs. 320
lakhs by the end of the year, assuming net income for the year will be Rs. 160 lakhs?

Answer
a) Important Ratios used in different situations
(i) Liquidity Ratios –
Liquidity or short term solvency ratios would be used by the bank to check the ability of the company to repay its short-
term liabilities. A Bank may use current ratio or Quick ratio to judge short term solvency of the company. Further interest
coverage ratio shall also be analysed to ensure the interest repayment security.
(ii) Capital Structure or Leverage Ratios –
The long-term creditor would use the capital structure or leverage ratios to ensure the long term stability and structure of
the firm. A long term creditor interested in determining whether his claim is adequately secured may use Debt-servicing
coverage and interest coverage ratio.
(iii) Profitability Ratios –
The shareholder would use the profitability ratios tomeasure the operational efficiency of the company to see the final
results of business operations. A shareholder may use return on equity, earning per share and dividend per share ratios.
Price earning ratio and book value per share are also analysed to decide wheather a particular share is to sell or hold.
b)
i) M-M Approach of calculating share price:
P0= (P1+D1) / (1+Ke)
Where,
P0= Existing market price per share i.e. Rs. 120
P1= Market price of the share at the year end (to be determined)
D1= Contemplated dividend per share i.e. Rs. 6.40
Ke= Capitalization rate i.e. 9.6 %

Calculation of Share price when:


(i)Dividend is declared
P0= (P1+D1) / (1+Ke)
120 = (P1 + 6.4)/ (1+0.096)
P1= Rs. 125.12
(ii)Dividend is not declared
P0= (P1+D1) / (1+Ke)
120 = (P1 + 0)/ (1+0.096)
P1= Rs. 131.52

ii. Calculation of No. of shares to be issued:


Particulars If dividend is If dividend is not
declared declared
Nets Income (Rs. In Lakhs) 160 160
Less: Dividend (Rs. In Lakhs) 51.20 -
Retained Earnings (Rs. In Lakhs) 108.80 160
Investment budget (Rs. In Lakhs) 320 320
Amount to be raised by issue of shares (Rs. In Lakhs) 211.20 160
Market price per share (in Rs.) 125.12 131.52
No. of new shares to be issued ( in numbers) 1,68,798 1,21,655

3.
a) You are provided with the following extract of cost sheet of ABC Ltd:
Per unit (Rs.)
Raw material 50
Direct Labour 20
Overhead (including depreciation of Rs. 10) 40
Total Cost 110
Profit 20
Selling price 130

The Institute of Chartered Accountants of Nepal 28


Suggested Answers - Financial Management CAP-II Examination - December 2014

Average raw material in stock is for one month. Average material in work-in-progress is for half month. The suppliers allow
credit for one month to the company; and it also allows one month credit to its customers. Average time lag in payment of
wages and overheadsare 10 days and 30 days respectively. 25% of the sales are on cash basis. Cash balance is expected to be
Rs. 100,000. Finished goods are expected to lie in the warehouse for one month.Production is carried on evenly throughout the
year and wages and overheads accrue accordingly.

Required: 7

Prepare a statement of the working capital needed to finance the level of activity of 54,000 units of output. State your
assumptions, if any.

b) JK Ltd. has appointed you as its Finance Manager. The company wants to implement a project for which Rs. 60 lakh is
required to be raised from the market as a means of financing the project. The following financing plans and their options are at
hand:
(Rs. in lakh)
Particular Plan X Plan Y Plan Z
Option 1: Equity Shares 60 60 60
Option 2: Equity Shares 30 40 20
13% Preference Shares Nil 20 20
10% Non Convertible Debentures 30 Nil 20

Assume corporate tax rate to be 25 per cent, and the face value of all the shares and debentures to be Rs. 100 each.
Required: (7+1=8)
i) Calculate the indifference points and earnings per share (EPS) for each of the financing plan.
ii) Which plan should be accepted by the company? Why?

Answer
a) As the annual level of activity is given at 54,000 units, it means that the monthly would be 54,000/12=4,500 units. The monthly
working capital requirement for this monthly turnover can now be estimated as follows:
Estimation of Working Capital Requirement

I. Current Assets: Amount(Rs.)

Minimum Cash Balance 100,000


Inventories:
Raw Materials (4,500×Rs.50) 225,000
Work in Progress:
Materials (4,500× Rs. 50)/2 112,500
Wages 50% of (4,500× Rs. 20)/2 22,500
Overheads 50% of (4,500× Rs. 30)/2 33,750
Finished Goods (4,500× Rs.100) 450,000
Debtors(4,500× Rs.100×75%) (at cost) 337,500
Gross Working Capital 1,281,250

II. Current Liabilities:

Creditors for Materials(4500× Rs.50) 225,000


Creditors for Wages(4,500× Rs.20)/3 30,000
Creditors for Overheads(4,500× Rs. 30) 135,000
Total Current Liabilities 390,000
Net Working Capital (I-II) 891,250

Working Note/ Assumptions:

Note: Alternatively, student may assume debtors on selling price i.e. Rs. 130 in such a case , overhead needs to be calculated including
depreciation i.e. at Rs. 40 while calculating WIP & finished goods.
1. The Overheads of Rs. 40 per unit include depreciation Rs. 10 per unit, which is non-cash item. This depreciation cost has been
ignored for valuation of work-in-progress, finished goods and debtor. The overhead cost, therefore, has been taken only at Rs.
30 per unit.
2. In the valuation of work-in-progress, the raw materials have been taken at full requirements for 15 days; but the wages and
overheads have been taken only at 50% on the assumption that on an average all units in work in progress are 50% complete.

The Institute of Chartered Accountants of Nepal 29


Suggested Answers - Financial Management CAP-II Examination - December 2014

3. Since, the wages are paid with a time lag of 10 days, the working capital provided by wages have been taken by dividing the
monthly wages by 3 (assuming a month to consist of 30 days)

b)
i) Determination of indifference point under plans X, Y, Z
Let X be the EBIT in all cases.
Plan X : X(1-t)/N1 = (X-Interest)(1-t)/N2
Or,X (1-0.25)/60,000 = (X-300,000)(1-0.25)/30,000
Or, X-0.25X=2(0.75X-225,000)
Or, X-0.25X=1.5X-450,000
Or, 0.75X=Rs. 450,000
Or, X = Rs. 600,000
Plan Y : X(1-t)/N1 = [(X-Interest)(1-t)-Dp]/N2
Or, X (1-0.25)/60,000 = [X(1-0.25)-260,000]/40,000
Or, 0.75X/60,000=(0.75X-260,000)/40,000
Or, 2(0.75X)=3(0.75X-260,000)
Or, 1.50X=2.25X-780,000
Or, X = 780,000/0.75
Or, X = Rs. 1,040,000
Plan Z : X(1-t)/N1 = [(X-Interest)(1-t)-Dp]/N2
Or, X (1-0.25)/60,000 = [(X-200,000)×(1-0.25)-260,000]/20,000
Or, 0.75X/60,000=(0.75X-410,000)/20,000
Or, 0.75X=3(0.75X-410,000)
Or, 0.75X=2.25X-1,230,000
Or, X = Rs. 820,000
Determination of EPS under Plans X , Y and Z for options 1 and 2

Plan X Plan Y Plan Z


Particulars
1 2 1 2 1 2
EBIT 600,000 600,000 1,040,000 1,040,000 820,000 820,000
Less : Interest - 300,000 - - - 200,000
EBT 600,000 300,000 1,040,000 1,040,000 820,000 620,000
Less : Taxes @ 25% 150,000 75,000 260,000 260,000 205,000 155,000
EAT 450,000 225,000 780,000 780,000 615,000 465,000
Less: Preference Dividend - - - 260,000 - 260,000
Earnings available to Equity holders 450,000 225,000 780,000 520,000 615,000 205,000
No. of Equity Shares 60,000 30,000 60,000 40,000 60,000 20,000
EPS 7.50 7.50 13 13 10.25 10.25

ii) The company should adopt Plan Y since the EPS is maximum under this plan.

4.
a) Consider from the following information:

Equity share capital (Rs.100 each) Rs. 5,000,000


Reserves and surplus Rs.500,000
15% secured loans Rs. 2,500,000
12.5% unsecured loans Rs. 1,000,000
Fixed assets Rs. 3,000,000
Investments Rs.500,000
Operating profit Rs. 2,500,000
Tax rate 25%
PE ratio 12.5
Required: 5
Calculate the value of each equity share.

The Institute of Chartered Accountants of Nepal 30


Suggested Answers - Financial Management CAP-II Examination - December 2014

b)
i) Acompany has to make the payment of Rs. 2,000,000 on 5th of March 2015. It has some surplus moneytoday i.e.,4th
December 2014 and it has decided to invest in a deposit of bank at 8% per annum to meet the amount for payment. What
money is required to be invested now? Take year as 365 days. 2
ii) Calculate the market sensitivity index and the expected return on investment from the following data: 3

Standard deviation of an asset 2.5%


Market standard deviation 2%
Risk free rate of return 13%
Expected return on market portfolio 15%
Correlation coefficient of portfolio with market 0.8

c) ABC limited expects, with some degree of certainty, to generate the following profits and to have the following capital
investment during the next five years.
(Rs. in thousand)
Year 1 2 3 4 5
Net Income 5,000 4,000 2,500 2,000 1,500
Investment 2,500 2,500 3,200 4,000 5,000

The investments are financed first from the same year profit and the shortfall,if any, shall be externally financed.
The company currently has 1,000,000 equity shares and pays dividend of Rs. 5 per share.
Required:
i) Determine dividend per share, if dividend policy is treated as a residual decision. 2
ii) Determine dividend per share and the amount of the external financing that will be necessary, if a dividend payout ratio of
50% is maintained. 3
Answer
a) We have,
Value= EPS × PE Ratio
EPS calculation Rs.
Operating profit i.e. EBIT 2,500,000
Less: Interest on 15% secured loans 375,000
Interest on 12.5% unsecured loans 125,000
Profit before tax 2,000,000
Tax @ 25% 500,000
PAT 1,500,000
Number of equity shares = (Rs 5,000,000/100) = 50,000
Therefore EPS = (Rs 1,500,000/50,000)= Rs. 30
P/E Ratio is given as 12.5
Therefore,
Value of equity share =EPS×PE Ratio= 30×12.5 =Rs.375

b)
i) Target money: Rs. 2,000,000
The amount to be invested now is in fact the present money of this targeted money. The FVF may be ascertained as follows:
FVF = 1+ [(annual rate of interest × (Deposit period/365)]
=1+[(0.08x(90/365)]
=1.01972
Now the present value of the target amount can be ascertained as follows:
Present value = Target amount/1.01972
= Rs. 1,961,323

Note: Student may use PVF and in such case present value = Target money×PVF
the deposit of Rs. 1,961,323 at the rate of 8% for a period of 90 days will accumulate to Rs. 2,000,000. Therefore, amount to be
invested is Rs. 1,961,323

ii) Market sensitivity index i.e. Beta factor can be calculated as follows:
Βeta = (Standard Deviation of asset/Standard Deviation of Market) ×Correlation Coefficient of Market
= (2.5/2)×0.80
=1

The Institute of Chartered Accountants of Nepal 31


Suggested Answers - Financial Management CAP-II Examination - December 2014

Now, the expected return on the investment canbe ascertained with the help of CAPM equation as follows:
R =rf+(Rm-rf)β
=13%+(15-13%)1
=15%

c)
i) Calculation of the Dividend per share if dividend policy is treated as a residual decision

Year Profit(Rs.) Investment(Rs.) Balance(Rs.) DPS(Rs.)


1 5,000,000 2,500,000 2,500,000 2.50
2 4,000,000 2,500,000 1,500,000 1.50
3 2,500,000 3,200,000 - 0
4 2,000,000 4,000,000 - 0
5 1,500,000 5,000,000 - 0

ii) Calculation of the Dividend per share and external financing required at 50% payout
External
Year Profit(Rs.) Dividends(Rs.) DPS(Rs.) Investment(Rs.) Financing(Rs.)

1 5,000,000 2,500,000 2.50 2,500,000 -

2 4,000,000 2,000,000 2.00 2,500,000 500,000

3 2,500,000 1,250,000 1.25 3,200,000 1,950,000

4 2,000,000 1,000,000 1.00 4,000,000 3,000,000

5 1,500,000 750,000 0.75 5,000,000 4,250,000

5.
a) Following is the data regarding six securities:

Securities A B C D E F
Return % 10 10 15 5 11 10
Risk (SD) % 5 6 13 5 6 7

Required: (2+4=6)
i) Which of three securities will be selected by an investor and why?
ii) Assuming perfect negative correlation, analyse whether it is preferable to invest 80% of money in Security A and 20% in
Security C or to invest 100% of money in Security E.
b) A firm is contemplating to increase its credit period from 30 days to 60 days. The average collection period, which is at present
45 days, is expected to increase to 75 days. Due to this change, the bad debt expenses is expected to increase from the current
level of 1 percent to 3 percent of sales. Total credit sales are expected to increase from the level of 30,000 units to 34,500 units.
The present average cost per unit is Rs. 8. The variable cost and sales are Rs. 6 and Rs. 10 per unit respectively. The firm
expects a rate of return of 15 percent.

Required: 5
Analyse the firm's proposal to change the credit period and advise.
c) Discuss about the fundamental principles governing capital structure. 4

The Institute of Chartered Accountants of Nepal 32


Suggested Answers - Financial Management CAP-II Examination - December 2014

Answer
a)
i) Arranging the securities in the increasing order of risk:
Risk % Return % Security Invest
5 10 A Yes
5 5 D No
6 10 B No
6 11 E Yes
7 10 F No
13 15 C Yes

Securities A, E and C should be selected. The investor will try to minimize risk and maximize return. Therefore, security A is
better than D, security E is better than B and C is better with highest return.

(ii) Calculation of overall return and overall standard deviation (Risk) when 80% in Security A and 20% in Security C is invested.
Overall return = ReturnA×WeightA+ ReturnC×WeightC
= 10%×0.8 + 15%×0.2
=11%
Overall Variance (SD2) = (SDA× WA)2 + (SDC× WC)2 + 2× (SDA× WA) × (SDC× WC) ×rAC
SD2 = (5×0.80)2 + (13×0.20)2 + 2 ×(5×0.8) × (13×0.2) × (-1)
2
SD = 16 + 6.76 + 2 X 4 X 2.6 X (-1)
= 22.76 - 20.8
= 1.96
SD = 1.4
Summary
Alternative-1 Alternative-2 (100%
(80% in A and 20% in C) in E)
SD (%) 1.4 6
Overall return (%) 11 11

Conclusion:
Since same return i.e. 11% is available from lower risk (SD) of 1.4 %, Alternative-1 is preferable over alternative-2.

b) Profit on additional sales =Rs.(10 –6) × (34,500 -30,000)units


= 4 ×Rs. 4,500
= Rs. 18,000

Average Investment in Accounts Receivable:


Present = (Total Sales Qty ×Average Cost Per Unit)/Receivable Turnover ratio
= (30,000×8) / (360/45)
= 240,000/8
=Rs.30,000
Proposed = (Existing Cost of sales + additional cost of sales)/Receivable Turnover ratio
= (2,40,000+4,500×6) / (360/75)
= 2,67,000 / 4.8
= Rs. 55,625
Additional Investment in Accounts Receivable = Rs. 55,625 – Rs. 30,000
= Rs. 25,625
Cost of additional investment in Accounts Receivable at 15% = Rs. 25,625× 15%
= Rs. 3,843.75
Additional Bad debt expenses
= (34,500×10×0.03) – (30,000×10×0.01)
= 10,350 – 3,000
= Rs. 7,350
The Net Effect of Proposed increase in credit period
= Additional Profit – additional expenses
= Rs. 18,000 – Rs.3,843.75 – Rs. 7,350
= Rs. 18,000 – Rs. 11,193.75
= Rs. 6,806.25

Conclusion:
The extension of credit period would result in net gain of Rs. 6,806.25, so the firm is advised to extend credit period from
30 days to 60 days.

The Institute of Chartered Accountants of Nepal 33


Suggested Answers - Financial Management CAP-II Examination - December 2014

c) The fundamental principles governing capital structure are:


(i) Cost Principle –
This principle suggests that an ideal capital structure is one that minimizes cost of capital structure and maximizes earning
per share.
(ii) Risk Principle –
This principle suggests using more proportion of common equity for financing requirement. Use of more and more debts
means higher commitment in form of interest payout. This would lead to erosion of shareholders value in unfavorable
business situation.
(iii) Control Principle –
This principle suggests to consider interest of maintain existing management and operational control over the company.
Management may wish to have control undisturbed.
(iv) Flexibility Principle –
This principle gives flexibility to the management who decides such a combination of sources of financing which it finds
easier to adjust according to changes in need of funds in future too.
(v) Other considerations –
Besides above principles, other factors such as regulatory requirements, nature of industry, timing of issue and
competition in the industry should also be considered.

6. Write short notes on: (4×2.5=10)


a) Profitability Index
b) Treasury Bills
c) Limitations of profit maximization objective of Financial Management
d) Capital Rationing
Answer
a) Profitability index is an investment appraisal technique calculated by dividing the present value of future cash flows of a
project by the initial investment required for the project.
Profitability Index is calculated as follows:
Present Value of Future Cash Flows
PI =
Initial Investment Required
Profitability index is actually a modification of the net present value method. While present value is an absolute measure (i.e. it
gives as the total figure for a project), the profitability index is a relative measure (i.e. it gives as the figure as a ratio).

The decision rule is to accept a project if the profitability index is greater than 1, stay indifferent if the profitability index is
zero and don't accept a project if the profitability index is below 1.

Profitability index is sometimes called benefit-cost ratio too and is useful in capital rationing since it helps in ranking projects
based on their per dollar return.

b) Treasury bills are obligation of the government. They are sold on discounted basis. The investor does not receive an actual
interest payment. The return is the difference between the purchase price and the face (par) value of the bill.

The treasury bills are issued only in bearer form. They are purchased, therefore, without investors' name upon them. This
makes them easily transferable from one investor to another. A very active secondary market exists for these bills. The
secondary market for bills not only makes them highly liquid but also allows purchase of bills with very short maturities. As
the bills have the full financial backing of the government, they are, for all practical purposes, risk-free. The negligible
financial risk and the high degree of liquidity make their yield lower than those on the other marketable securities. Due to their
virtually risk free nature and because of active secondary market for them, treasury bills are one of the most popular marketable
securities even though the yield on them is lower.

c) Profit maximization objective of financial management has following limitations:


¾ It ignores time factor.
¾ It ignores the risk factors.
¾ It focuses on short term profits and overlooks long term vision.
¾ This is vague because it is not clear whether the term relates to economic profit, accounting profit, profit after tax or
before tax.
¾ The term profit is also ambiguous.

d) Capital Rationing is the process hereby the limited funds available are allocated amongst the financially viable projects which
are not mutually exclusive under consideration so as to maximize the wealth of the shareholders. Thus, capital rationing
situation is said to exist if:
i. Limited funds are available for investment.
ii. More than one financially viable projected which are not mutually exclusive are under consideration.

The Institute of Chartered Accountants of Nepal 34


Suggested Answers - Financial Management CAP-II Examination - December 2014

7. Distinguish between: (4×2.5=10)


a) Capital structure and Financial structure b) Clean packing credit and Packing credit against hypothecation of goods
c) Capital market and Money market d) Systematic risk and Unsystematic risk
Answer
a) Capital Structure is the permanent long term financing of the company including Long term debt, equity capital, preferential
shares and retained earnings is called capital structure. It can be also termed as a mix of long term finances used by the
company.It is the financing plan of the company. It differs from Financial Structure which includes short term debt and
accounts payable also.
Whereas, financial Structure entails the ways the assets of the companies are financed such as trade accounts payable, short
terms borrowings as well as long term borrowings and ownership equity. Financial structure is distinguished from capital
structure where only long term debt and equity are included. A company’s financial structure is influenced by many factors
such as growth rate, stability of sales. It is the basic frame of references for analysis concerned with financial leveraging
decisions.
b) Packing Credit is an advance extended by banks to an exporter for the purpose of buying, manufacturing, processing, packing,
shipping goods to overseas. If an exporter has a firm export order placed with him by his foreign customer (buyer) or all
irrevocable Letter of Credit in his favour, he can approach a Bank for Packing Credit Facility.
Clean Packing Credit
• This facility is extended only on production of a firm export order or a letter of credit.
• There is no charge or control over raw material or finishes goods that constitute the supply.
• The bank takes into consideration trade requirements, credit worthiness of exporter and its margin.
• Export Credit Guarantee Corporation (ECGC) insurance cover should be obtained by the bank.
Packing credit against hypothecation of goods
• This facility is extended only on production of a firm export order or a letter of credit.
• The goods which constitute the supply are hypothecated to the Bank as security with stipulated margin.
• The goods shall be exported by the borrower. The Bank does not have any effective possession of the same.
• The exporter has to submit stock statements at the time of sanction and also periodically and for whenever there is any
movement in stock.

c) Capital market and money market are two basic components of financial system. Capital market deals with long and medium
term instruments of financing while money market deals with short term instruments. Capital market instruments are shares,
debentures, mutual funds etc. while money market instruments are interbank placement, call money, commercial papers,
treasury bills etc.
Capital market is usually classified as primary market and secondary market while there is no such classification of money
market.
Capital market participants include retail investors, institutional investors, financial institutions, corporate houses and banks
while money market participants include banks, financial institutions, central bank and government.
d)
Systematic Risk Unsystematic Risk
A part of the risk that arises on account of the A part of the risk that arises from uncertainties which are
economy-wide uncertainties and the tendency of unique to individual securities, and which are diversifiable if
individual securities to move together with changes large number of securities is combined to form well-
in the market. This part of risk cannot be reduced diversified portfolios. The unique risks of individual securities
through diversification, and it is called systematic or in a portfolio cancel out each other. This part of the risk can
market risk. be totally reduced through diversification and is called
unsystematic or unique risk.
Investors are exposed to market risk even when they
hold well-diversified portfolios of the securities.
Examples of systematic risk are: Examples of unsystematic risk are:
• The Government changes the interest rate • workers declare strike in a company
policy • the R&D expert of the company leaves
• The corporate tax rate is increased • a formidable competitor enters the market
• The Government resorts to massive deficit • the company loses a big contract in a bid
financing • the company makes a breakthrough in process innovation
• The inflation rate increases • the Government increases custom duty on the material used
• The Nepal Rastra bank promulgates a by the company
restrictive credit policy • the company is not able to obtain adequate quantity of raw
materials from the suppliers.

The Institute of Chartered Accountants of Nepal 35


Suggested Answers - Cost & Management Accounting CAP-II Examination - December 2014

Cost & Management Accounting

1. A company markets products X and Y which it makes by using its capacity to the extent of 50% on X and 30% on Y. Budget for
2014 is as given below:
X Y
Production units 5,000 4,500
Direct materials/unit Rs. 300 Rs. 200
Conversion cost/unit
Variable Rs. 100 Rs. 80
Fixed Rs. 50 Rs. 40

Selling price per unit Rs. 500 Rs. 350


Profit per unit Rs. 50 Rs. 30

For the next year's budget, the following factors are relevant:
ƒ Direct Material cost will go up by 6%
ƒ Variable conversion cost will increase by 10%
ƒ Selling price of: X will be increased by 4% and Y will be increased by 6%

To utilize the idle capacity of 20%, three proposals as under were put forth:

a) Produce X and sell the output at the revised price. Production of X from the idle however will be less by 10%
b) Produce Y but the increased production will be sold at the existing price
c) Utilize the idle capacity to produce a new product Z whose details are as under:

ƒ Production form idle capacity: 2,000 units


ƒ Direct Materials: Rs.400 per unit
ƒ Variable conversion cost: Rs.200 per unit
ƒ Selling price: Rs.700 per unit
ƒ Special publicity Expense: Rs.20,000
ƒ The present allocation of 50% and 30% respectively on products X and Y cannot be changed.
You are required to prepare Statements of Profitability for 2014 and 2015. 20

Answer:

Statement showing the profitability of the products for 2014 as per budget production (units)
X= 5,000 and Y=4,500
Details Per unit Total Total
X Y X Y
1.Selling Price 500 350 2,500,000 1,575,000 4,075,000
2.Costs:
Direct Material 300 200 1,500,000 900,000 2,400,000
Conversion Cost:
Variable 100 80 500,000 360,000 860,000
Fixed 50 40 250,000 180,000 430,000
3.Total of 2 450 320 2,250,000 1,440,000 3,690,000
4.Profit (1-3) 50 30 250,000 135,000 385,000

a) Total contribution if Product X is produced


Production of X at 50% Capacity 5,000 units
Production of X at 20% Capacity 2,000 units
Less: 10% of this production 2,00 units
Net production of X if idle capacity is utilized 1800 units

The Institute of Chartered Accountants of Nepal 36


Suggested Answers - Cost & Management Accounting CAP-II Examination - December 2014

Existing Increase Revised


(%) (Per unit)
Selling price 500 4% 520
Direct Materials 300 6% 318
Conversion Costs –variable 100 10% 110
Total Variable Cost 428
Contribution per unit 92
Total Contribution (1,800* Rs.92) Rs.165,600

b) Total contribution if Product Y is produced


Production of Y at 30% Capacity 4,500 units
Production of Y at 20% Capacity 3,000 units

Existing Increase Revised


(%) (Per unit)
Selling price 350 350
Direct Materials 200 6% 212
Conversion Costs –variable 80 10% 88
Total Variable Cost 300
Contribution per unit 50
Total Contribution (3,000* Rs.50) Rs.150,000

c) Total contribution if a new product Z is produced


(Per unit)
Selling price Rs.700
Direct Materials Rs.400
Variable conversion cost Rs.200
Total Variable Cost Rs.600

Contribution per unit Rs.100

Total contribution ( 2,000 units * Rs.100) Rs.200,000


Less: Special publicity expense Rs.20,000

Net Total Contribution Rs.180,000

This proposal should be accepted as the total contribution is highest when product Z is produced.

It is given in the proposal ( b) above that if product Y is produced in the idle time, then additional production will be sold at the
existing prices. Since proposal (c) is recommended, the revised selling price of product Y will be:

Existing selling price per unit Rs.350


Add: 6% increase in price Rs.21
Revised price of product Y Rs.371

Statement showing the profitability of three products for 2015 as per revised budget
production (units) X= 5,000, Y=4,500 and Z=2,000
Details Per unit Total Total
X Y Z X Y Z
1.Selling Price 520 371 700 2,600,000 1,669,500 1,400,000 5,669,500
Direct Material 318 212 400 1,590,000 954,000 800,000 3,344,000
Conversion Cost- 110 88 200 550,000 396,000 400,000 1,346,000
variable
Total V. costs 428 300 600 2,140,000 1,350,000 1,200,000 4,690,000
Contribution 92 71 100 460,000 319,500 200,000
Total Contribution 979,500
Less: Fixed Cost 430,000
Publicity for Z 20,000
Net Profit 529,500

The Institute of Chartered Accountants of Nepal 37


Suggested Answers - Cost & Management Accounting CAP-II Examination - December 2014

2.
a) The following information is available from the financial books of a company having a normal production capacity of 60,000
units for the year ended 31st March, 2014:

• Sales Rs. 10,00,000 ( 50,000 units)


• There was no opening and closing stock of finished units.
• Direct Material and direct wages cost were Rs. 5,00,000 and Rs. 2,50,000 respectively.
• Actual factory expenses were Rs. 1,50,000 of which 60% are fixed.
• Actual administrative expenses were Rs. 45,000 which are completely fixed.
• Actual selling and distribution expenses were Rs. 30,000 of which 40% are fixed.
• Interest and dividends received Rs. 15,000.

You are required to: (3+4+3=10)


st
i) Find out profit as per financial books for the year ended 31 March, 2014.
ii) Prepare the cost sheet and ascertain the profit as per cost accounts for the year ended 31st March 2014 assuming that the
indirect expenses are absorbed on the basis of normal production capacity; and
iii) Prepare a statement reconciling profit shown by financial and cost books.

b) Shakti Engineers are engaged in construction and erection of a bridge under along-term contract. The cost incurred up to
31.03.2014 was as under:

Fabrication Rs.InLakhs
Direct Material 280
Direct Labour 100
Overheads 60
440
Erection cost to date 110
550
The contract price is Rs.11 crores and the cash received on account till 31.03.2014 was Rs.6 crores.
The technical estimate of the contract indicates the following degree of completion of work. Fabrication: Direct
Material 70%, Direct Labour and Overheads 60% Erection 40%.
You are required to estimate the profit that could be taken to Profit and Loss Account against this partly completed contract
as at 31.03.2014. 10

Answer 2 (a) (i)


As per Financial Books
Profit and Loss Account
(for the year ended 31st March 2014)

To Direct Material Rs. 5,00,000 By Sales ( 50,000 units) Rs. 10,00,000


To Direct Wages 2,50,000 By Interest and dividend 15,000
To Factory Expenses ( Actual) 1,50,000
To Admn. Exenses 45,000
To Selling and Distribution 30,000
To Profit 40,000 _____________
10,15,000 10,15,000
As per above account, profit is Rs. 40,000 for the year ended 31st March, 2014.

Answer 2 (a) (ii)


Cost Sheet
( for the year ended 31st March, 2014)
_________________________________________________________________________________
Normal Production capacity (units) 60,000
Sales/ Production (units) 50,000

Direct material Rs. 5,00,000


Direct wages 2,50,000
Prime cost 7,50,000
Factory overhead - Variable Rs. 60,000
- Fixed Rs. 90,000 X 5/6 75,000 1,35,000

The Institute of Chartered Accountants of Nepal 38


Suggested Answers - Cost & Management Accounting CAP-II Examination - December 2014

Works cost 8,85,000


Administrative expenses Rs. 45,000 X 5/6 37,500
Total cost of production 9,22,500
Selling and distribution expenses
- Variable Rs. 18,000
- Fixed Rs. 12,000 X 5/6 10000 28,000
Cost of Sales 9,50,500
Profit (balance) 49,500
Sales 10,00,000

Answer 2 (a) (iii)


Reconciliation Statement
Profit as per Cost Accounts Rs. 49,500
Add: Income from dividend ( not considered in Cost Accounts) 15,000 64,500

Less: Expenses undercharged in Cost Accounts:


i) Factory expenses ( 1,50,000-1,35,000) 15,000
ii) Adm. Expenses ( 45,000-37,500) 7,500
iii) Selling & Distribution ( 30,000-28,000) 2,000 24500
Profit as per financial accounts 40,000

Answer 2 (b)
EstimationofProfittobetakentoProfitandLossAccountagainstpartlycompletedcontractasat31.03.2014.
Profit to be taken to P/L Account= 2/3 X Notional profit X Cash received
Work certified
(Refer to working notes 1,2,3,&4)
= 2/3 X Rs. 92.48 lakhs X Rs. 600 lakhs
Rs. 642.48 lakhs
= Rs. 57.576 lakhs
Working Notes:
1. Statement showing estimated profit to date and future profit on the completion of contract

Particulars Cost to date Further Cost Total Cost

% Amount % Amount
Completion Rs. Completion Rs. Rs.
To date (a) to be done (b) (a)+(b)
Direct Material 70 280.00 30 120.00 400.00
Directlabour 60 100.00 40 66.67 166.67
Overheads 60 60.00 40 40.00 100.00
TotalFabricationcost(A) 440.00 226.67 666.67
Erectioncost:(B) 40 110.00 60 165.00 275.00
Totalestimatedcosts:(A+B) 550.00 391.67 941.67
Profit(Refertoworkingnote2) 92.48 65.85 158.33
642.48 457.52 1,100.00

2. Profit to date (notional profit) and future are calculated as below:


Profit to date (notional profit) = Estimated profit on whole contract X cost to date
Total Cost
= Rs. 158.33 XRs. 550
Rs. 941.67
= Rs. 92.48 ( Lakhs)

Future Profit = Rs. 158- Rs. 92.48


= Rs. 65.85

The Institute of Chartered Accountants of Nepal 39


Suggested Answers - Cost & Management Accounting CAP-II Examination - December 2014

3. Work certified
= cost of the contract to date + profit to date
= Rs. 550 + Rs. 92.49 = Rs. 642.48 lakhs

4. Degree of completion of contract to date


= Cost of the contract to date X 100
Contract Price
= Rs. 642.48 lakhs X 100
Rs. 1,100 lakhs
= 58.40%

3.
a) MTC Limited uses chemicalX in one of its finished products. The chemical-X is purchased from a vendor outside Nepal.
MTC Limited purchases 36,000 Ltr. of chemicalX per year at the rate of Rs. 900 per Ltr plus import duty @10% on such
purchases.

The chemicalX is used evenly throughout the year in the production process on a 360dayperyear basis. The Company incurs
Rs. 1,75,000 on one year agreement for material supply with the vendor and it estimates that Rs. 35,000 will be incurred to
place a single purchase order. The chemicalX is needed to be kept in a very carefully controlled temperature and humidity
conditions. MTC Ltd. Incurs 1.5% and 0.2676% of the value of inventory as storage cost and as insurance cost respectively.

Delivery from the vendor generally takes 12 days, but it can take as much as 16 days. The days of delivery time and
percentage of their occurrence are shown in the following tabulation:

Delivery time (days) : 12 13 14 15 16


Percentage of occurrence : 70 10 10 5 5
Required: (3+3+4=10)
i) Compute the economic order quantity (EOQ).
ii) Assume the company is willing to assume a 10% risk of being out of stock. What would be the safety stock? The re-
order point?
iii) Assume 5% stock-out risk. What would be the total cost of ordering and carrying inventory for one year?

b) Aryan Ltd. has three production department M,N and O and the two service department P and Q.
The following particulars are available for the month of September 2013:

(Rs.)
Lease rental 35,000
Power and Fuel 4,20,000
Wages to factory supervisor 6,400
Electricity 5,600
Depreciation on machinery 16,100
Depreciation on building 18,000
Payroll expenses 21,000
Canteen expenses 28,000
Provident fund contribution 58,000

Following are the further details available:

Particulars M N O P Q
Floor space (Sq.M) 1,200 1,000 1,600 400 800
Light Points(Nos) 42 52 32 18 16
Cost Of machine(Rs.) 12,00,000 10,00,000 14,00,000 4,00,000 6,00,000
No of Employees (Nos) 48 52 45 15 25
Direct wages (Rs.) 1,72,800 1,66,400 1,53,000 36,000 53,000
HP of Machines 150 180 120 - -
Working Hours(hours) 1,240 1,600 1,200 1,440 1,440

The Institute of Chartered Accountants of Nepal 40


Suggested Answers - Cost & Management Accounting CAP-II Examination - December 2014

The Expenses of service department are to be allocated in the following manner:

M N O P Q
P 30% 35% 25% - 10%
Q 40% 25% 20% 15% -

You are required to calculate the overhead absorption rate per hour in respect of the three production departments. 10
Answer
3 a)
(i) Economic Order Quantity (E.O.Q)
= √2AO = √2x36,000 Ltrs. xRs 35000 = 12,000 litres
C Rs 17.50
(ii) Safety Stock at 10% risk of being out of stock
Safety Stock required for two days i.e. for 13th and 14th day

Safety stock = 36,000ltr x 2days = 200 litres


360 days
Re-order Point = Minimum Stock level + Average lead time x Average consumption
= 200+12 x 100
= 1400 litres
(iii) At 5% risk of being out stock, safety stock will be safety stock for three days
100 ltr x 3 days =300 ltr.

Total Ordering Cost =36,000ltr x Rs 35,000 = Rs 1,05,000


12,000ltr

Total Carrying cost of inventory = (Safety Stock + Average inventory) Carrying Cost per litre per annum
= (300 + 1/2 x 12,000 ltr.) Rs 17.50
= Rs 1,10,250
Total cost of ordering & carrying inventory = 1,05,000 + 1,10,250 = Rs 2,15,250

Working Notes

1. Risk of being out of stock

Percentage of Occurrence Cumulative percentage of Risk of non Occurrence


Delivery Time
(%) Occurrence (%) (%)

12 days 70 70 30

13 days 10 80 20

14 days 10 90 10

15 days 5 95 5

16 days 5 100 0

2 (a) Ordering Cost per order (O)- Rs 35,000


(b) Cost per litre of chemical-X
Rate per litre Rs 900
Add: Import duty @10% Rs 90
Rs 990
Carrying cost per litre per annum of chemical-X (C)
1.7676% (1.5%+0.2676%) of Rs 990= Rs17.50
(Note: Amount of Rs. 175,000 incurred on making agreement for material supply will be apportioned over the entire quantity
of 36,000 ltr and included with cost of chemical X. However for the purpose of calculating carrying cost i.e storage cost and
insurance cost only invoice cost of material is taken. Invoice cost consist of cost per litre of chemical X plus import duty.)

The Institute of Chartered Accountants of Nepal 41


Suggested Answers - Cost & Management Accounting CAP-II Examination - December 2014

3(b)

Item of cost Basis of apportion ment Total Production Department Service Department
(Rs)
M N O P Q
(Rs) (Rs) (Rs) (Rs) (Rs)
Lease Rental Floor space 35,000 8,400 7,000 11,200 2,800 5,600
(6:5:8:2:4)
Power & Fuel HP of machines x 4,20,000 1,26,408 1,95,728 97,864 - -
Working hours
(93:144:72)
Supervisor’s Working hours 6,400 1,964 2,535 1,901 - -
wages* (31:40:30)
Electricity Light points 5,600 1,470 1,820 1,120 630 560
(21:26:16:9:8)
Depreciation Value of machinery 16,100 4,200 3,500 4,900 1,400 2,100
On Machinery (6:5:7:2:3)
Depreciation Floor space 18,000 4,320 3,600 5,760 1,440 2,880
on building (6:5:8:2:4)
Payroll No Of Employes 21,000 5,448 5,903 5,108 1,703 2,838
Expenses (48:52:45:15:25)
Canteen No Of Employes 28,000 7,625 7,870 6,811 2,270 3,784
expenses (48:52:45:15:25)
PF Direct wages 58,000 17,244 16,606 15,268 3,593 5,289
contribution (864:832:765:180:265)
Total 608,100 176,719 244,562 149,932 13,836 23,051

*wages to supervisor is to be distributed to production departments only.


Since the service department has incurred direct wages, it has also to be allocated to the production department. Therefore , the process of
allocation is as under.

P Q
Apportioned Overhead 13,836 23,051
Add: Allocated direct wages 36,000 53,000
49,836 76,051

P = 49,836+ 0.15 Q …………..(I)


Q = 76,051+ 0.10 P……………(II)
Substituting the value of Q in (I) we get
P = 49,836 + 0.15 (76,051+ 0.10 P)
P = 49,836 + 1140Q +0.015 P
P = 61244/0985
P = Rs 62177
And Q = 76,051+0.10 x 62,177
=Rs 82,269
Secondary Distribution summary
Particulars Total M N O
Allocated and apportioned overheads as per 5,71,213 1,76,719 2,44,562 1,49,932
primary distribution
Add: 90% of P(30%,35% & 25%) 55,959 18,653 21,762 15,544
Add: 85% of Q(40%,25% & 20%) 69,928 32,907 20,567 16,454
2,28,279 2,86,891 1,81,930

Overhead rate per hour


M N O
Total Overhead cost (Rs.) 2,28,279 2,86,891 1,81,930
Workings hours 1,240 1,600 1,200
Rate per hour (Rs.) 184.09 179.31 151.61

The Institute of Chartered Accountants of Nepal 42


Suggested Answers - Cost & Management Accounting CAP-II Examination - December 2014

4.
a) Pawanputra Air owns single jet aircraft and operates between Kathmandu and Delhi only. Flights leave Kathmandu on
Mondays and Thursdays and depart from Delhi on Wednesdays and Saturdays. Pawanputra Air cannot afford any more flights
between Kathmandu and Delhi. Only tourist class seats are available on its flights and all tickets are booked by travel agents.
The following informations are collected.

Seating capacity per plane 360


Average passengers per flight 200
Flights per week 4
Flights per year 208
Average one way fare Rs. 5,000
Variable fuel cost Rs. 1,40,000 per flight
Food service to passengers (not charged to Passengers) Rs. 200 per passenger
Commission to travel agents 8% of fare
Fixed annual lease cost allocated to each flight Rs. 5,30,000 per flight
Fixed ground services (maintenance, check in,
Baggage handling cost) allocated to each flight Rs. 70,000 per flight
Fixed salaries of flight crew allocated to each flight Rs. 40,000 per flight
For the sake of simplicity assume that fuel cost are unaffected by the actual number of passengers on a flight.

Required: (5+3+2=10)
i) What is the operating income that Pawanputra Air makes on each way flight between Kathmandu and Delhi?
ii) The market research department of Pawanputra Air indicates that lowering the average one way fare to Rs. 4,800 will
increase the average number of passenger per flight to 212. Should Pawanputra Air lower its fare?
iii) Gem Travels, a tour operator approaches Pawanputra Air to charter its jet aircrafttwice each month, first to take Gem’s
international tourists from Kathmandu to Delhi and then bring the tourists back from Delhi to Kathmandu. If Pawanputra
Air accepts the offer, it will be able to offer only 184 (208 less 24) of its own flights each year. The terms of the charter
are:

(a) For each one-way flight Gem will pay Pawanputra Rs. 7,50,000 to charter the plane and to use its flight crew and
ground service staff.
(b) Gem will pay for fuel costs.
(c) Gem will pay for all food costs.

On purely financial considerations, should Pawanputra Air accept the offer from Gem Travels?

b) Two fitters, a labourer and a boy undertake a job on piece rate for Rs.1,290. The time spent by each of them is 220 ordinary
working hours. The rates of pay on time-rate basis are Rs.1.50 per hour for each of the two fitters, Re. 1 per hour for the
labourer and Re. 0.50 per hour for the boy.

Required: (6+2=8)
i) The amount of piece-work premium and the share of each worker, when the piece-work premium is divided
proportionately to the time wages paid.

ii) The selling price of the above job on the basis of the following additional data:
Cost of direct material Rs.2,010
Works Overhead at 20% of Prime Cost
Selling Overhead at 10% of Work Cost and
Profit at 25% on Cost of Sales.
c) Write short notes on the advantages of time rate remuneration plans. 2

The Institute of Chartered Accountants of Nepal 43


Suggested Answers - Cost & Management Accounting CAP-II Examination - December 2014

Answer 4(a)

i) Statement of operating income of Pawanputra Air for Kathmandu-Delhi flight (one way)

Fare received (per flight): 200 passengers x Rs. 5,000 Rs.10,00,000


Variable costs (per flight)
Fuel cost Rs. 1,40,000
Food (200 x Rs. 200) 40,000
Commission to Travel Agents (8% of Rs. 10,00,000) 80,000 Rs. 2,60,000
Contribution per flight Rs.7,40,000

Fixed cost (per flight)


Annual lease cost Rs. 5,30,000
Fixed ground service costs 70,000
Salaries of flight crew 40,000 Rs. 6,40,000
Operating income per flight 1,00,000

ii) Fare received (per flight): 212 passengers x Rs. 4,800 Rs.10,17,600
Variable costs (per flight)
Fuel cost Rs. 1,40,000
Food (212 x Rs. 200) 42,400
Commission to Travel Agents (8% of Rs. 10,17,600) 81,408 Rs. 2,63,808
Contribution per flight Rs.7,53,792

Excess contribution due to lowering of fare (Rs. 7,53,792 – Rs. 7,40,000) = Rs. 13,792
Pawanputra Air should lower its fare as it would increase its contribution by Rs. 13,792

iii) For financial considerations to decide whether Pawanputra Air should charter its plane to Gem Travels most profitable
option i.e. option (b) should be used.

Contribution of Pawanputra Air under option (b) Rs. 7,53,792


Pawanputra Air would get (per flight) for charter of plane Rs. 7,50,000

As there is a loss of Rs. 3,792 per flight Pawanputra Air should not accept the offer.
4 (b)

(i) Calculation of wages:


2 Fitters @ Rs. 1.50 per hour for 220hours each Rs. 660
1 Labourers @ Re. 1 per hour for 220 hours Rs. 220
1 Boy @ Re. 0.50 per hour for 220 hours Rs. 110
Total Rs. 990

Piece Work Premium


Total Wages agreed on piece-rate basis Rs. 1,290
Less: Wages calculated on time basis Rs. 990
Piece Work Premium Rs. 300

Amount of premium will be paid to workers in the ratio of 660:220:110 (or 6:2:1) as follows:
2 Fitters Rs.200.00
1 Labourer Rs. 66.67
1 Boy Rs. 33.33
Total Rs. 300.00

(ii) Computation of Selling Price: Rs.


Direct Material 2,010
Direct Wages 1,290
Prime Cost 3,300
Work Overheads at 20% in Prime Cost 660
Work Cost 3,960
Selling Expenses at 10% in Work Cost 396
Cost of Sales 4,356
Add: Profit at 25% on Cost of Sales 1,089
Selling Price 5,445

The Institute of Chartered Accountants of Nepal 44


Suggested Answers - Cost & Management Accounting CAP-II Examination - December 2014

4(c) The advantages of time rate remuneration plans are as follows:


(i) It is commonly recognized by all trade unions as well as worker .
(ii) It is a guaranteed income assured to the worker.
(iii) It is very easy to understand and simple to calculate the earnings of worker.
(iv) It involves less clerical work and detailed records are not necessary.
(v) Since the production is not the criteria for calculation of wages, tools and materials are handled carefully. Wastage is also
minimized.

5. Distinguish between: (4×2.5=10)


a) Controllable costs and Uncontrollable costs
b) Indifference point and Break-even Point
c) Production account and Cost Sheet
d) Bill of material and Material requisition note

Answer5(a)

Controllable costs and Uncontrollable costs

Controllable costs are the costs which can be influenced by the action of a specified member of an undertaking. Controllable costs
incurred in a particular responsibility centre can be influenced by the action of the executive heading that responsibility centre.

Uncontrollable costs are those costs which cannot be influenced by the action of a specified member of an undertaking. The
distinction between controllable and uncontrollable costs is not very sharp and is sometimes left to individual judgement.In Fact, no
cost is controllable; it is only in relation to a particular individual that we may specify a particular cost to be either controllable or
non-controllable.

Answer 5 (b)

Particulars Indifference Point Break-Even Point


Definition Indifference Point is the level of Sales at which BEP is the level of sales at which there isneither
Total costs and Profits of two options are equal. a Profit nor a Loss to thefirm. At BEP,
the total Contribution equals Fixed Cost.
Formula Indifference Point (in Rs.) =Difference in Fixed Break Even Point (in Rs.) =
CostDifference in Var. Cost ratio or PV ratio Fixed Cost
PV ratio

Significance It is the activity level at which Total Cost under It is the activity level at which the Total Revenue
two alternatives are equal. from a product mix is equal to its Total cost.
Purpose Used to choose between two alternative Used for profit planning.
options for achieving the same objective.

Answer 5(c)
The following are the points of difference between a Production Account and a Cost Sheet.
i) Production Account is based on double entry system whereas cost sheet is not based on double entry system.
ii) Production Account consists of two parts. The first part shows cost of the components and total productioncost. The second
part shows the cost of sales and profit for theperiod. Cost sheet presents the elements ofcosts in a classified manner and the cost
is ascertained at different stages such as prime cost; works cost ofproduction; cost of goods sold; cost of sales and total cost.
i) Production account shows the cost in aggregate and thus facilitates comparison with other financial accounts. Cost sheet shows
the cost in detail and analytical manner which facilitates comparison of costfor the purpose of cost control.
ii) Production account is not useful for preparing tenders or quotations. Estimated cost sheets can beprepared on the basis of actual
costs sheets and these are useful for preparing tenders or quotations.

The Institute of Chartered Accountants of Nepal 45


Suggested Answers - Cost & Management Accounting CAP-II Examination - December 2014

Answer 5(d)
Bill of material Material requisition note
It is a comprehensive list of materials with It is a formal written demand or request, usuallyfrom
exactdescription and specifications,required for a jobor the production department to store for thesupply of
other production units. This also providesinformation specified materials, stores etc. Itauthorises the
about required quantities so that ifthere is any deviation storekeeper to issue therequisitioned materials and
from the standards, it caneasily be detected. It is record the same onbin card.
prepared by theEngineeringor Planning Department in a
standard form.
The purpose of bill of material is to act as a The purpose of material requisition note is to
singleauthorisation for the issue of all materials drawmaterial from the store byconcerned departments.
andstores items mentioned in it. It provides anadvance
intimation to store department about therequirements of
materials. It reduces paper work.It serves as a work order
to the productiondepartment and a document for
computing thecost of material for a particular job or
work orderto the cost department.

6. Answer the following questions: (4×2.5=10)


a) Discuss the treatment of research and development expenditures in cost accounting.
b) The more kilometers you travel with your own vehicle, the cheaper it becomes. “Comment briefly on this statement”.
c) Discuss the accounting treatment for spoilage and defectives in cost-accounts
d) Enumerate the remedial steps to be taken to minimize the labour turnover.
Answer 6(a)
If research is conducted in the methods of production, the expenses should be charged to production overhead. If the research
relates to administration, the expenses arecharged to administration overheads. If it is related to market research, the expenses are
charged to S & D overheads. Development costs incurred in connection with a particular product should be charged directly to that
product. Such expenses are usually treated as deferred revenue expenditure and recovered as cost per unit of the product when
production is fully established. Routine nature research expenses are charged to general overheads.

Answer 6(b)
The cost per kilometre, (if one travels in his own vehicle) will decline when he travels more kilometers. This is because the
majority of costs for running and maintaining vehicles are of fixed and the component of fixed cost per kilometre goes on
decreasing with an increase in kilometre travel. Hence, the given statement is true.

Answer 6(c)
Normal spoilage (i.e. which is inherent in the operation) costs are included in cost either by charging the loss due to spoilage to the
production order or by charging it to production overhead so that it is spread over all the products. Any value realized from the sale
of spoilage is credited to production order or production overhead account, as the case may be. The cost of abnormal spoilage are
charged to Costing Profit & Loss Account.
Defectives that are considered inherent in the process and are identified as normal can be recovered by using any one of the
following method.
• Charged to good products
• Charged to general overheads
• Charged to departmental overheads.
If defectives are abnormal, they are to be debited to Costing Profit & Loss Account.

Answer 6(d)
The following steps are useful for minimizing labour turnover:
¾ Exit interview: An interview be arranged with each outgoing employee to ascertain the reasons of his leaving the organization.
¾ Job analysis and evaluation: to ascertain the requirement of each job.
¾ Organisation should make use of a scientific system of recruitment, placement and promotion for employees.
¾ Organisation should create healthy atmosphere, providing education, medical and housing facilities for workers.
¾ Committee for settling workers grievances.

The Institute of Chartered Accountants of Nepal 46


Suggested Answers - Business Communication CAP-II Examination - December 2014

Business Communication
Section -'A'

1. Read the following case carefully and answer the questions given below: (3.5+3.5+3=10)
Anand Roy, an officer at Research Division in Real Drinks Co. Ltd. is asked to prepare a report after conducting a nation-wide
survey about the sales condition of the products of the company. The Sales Manager has instructed Mr Roy to conduct the research
with a key intention of increasing the sales of ‘Real Drink’ in the coming summer. Other general aspects of the survey will be some
of those including the ways of increasing the demand of the product, improvements to be brought, desires of costumers, cost,
quantity, and marketing strategies, and so on. The survey should include at least ten thousand respondents across the country with
one thousand in each Development Region. Mr. Roy has to present some clear findings and recommendations in the report, and he
has now kept himself busy in developing research tools, sampling procedures, research design, analyzing measures, rating scales,
action plans, time table, etc.

Questions:
a) Choose a particular area of research, for example, marketing strategy, costumer desires, costumer perceptions, etc., and prepare
a set of questionnaire based on the chosen area in order to elicit information for the survey. The questionnaire should include at
least seven questions.
b) What could be three major objectives of the survey/report?
c) Prepare a timetable for writing the report that is required by the sales manager of the company.
Answer
a) Questionnaire on costumer perception on Real Drink(RD)
(To be asked to the costumers)
Questions Yes! Yes No No! Remarks
Do you love the flavor of Real Drink?
Do your friends love RD?
Do you prefer RD to other drinks?
Do you think quality of RD pays its cost?
Is chilled RD better than normal one?
Do you find RD in the parties too?
Can you imagine your lunch without RD?

b) The objectives of the study are:

i) To find out the ways of increasing the sales throughout the country,
ii) To find out the perception of consumers about the quality of the product,
iii) To find out the efficiency of marketing strategies of the company.

c) Time Schedule for the Survey


Actions/Activities Time allocated
Research tool design and planning 1 week
Survey procedure(sampling and data collection ) 4 weeks
Analysis and interpretation of data 2 weeks
Writing report 1 week
Submission after 8 weeks

2. As a manager of Express Welfare Organization, you are going to organize a blood donation program which is the part of company’s
annual activity. Since the people might become reluctant to participate in the program and donate the blood, you need to convince
them highlighting its importance and overcoming their possible objections. Now, write a persuasive letter to distribute the possible
donors in and outside the organization. Follow the techniques of writing persuasive message. 10

The Institute of Chartered Accountants of Nepal 47


Suggested Answers - Business Communication CAP-II Examination - December 2014

Answer
Express Welfare Organization
Baneshwor, Kathmandu

November 23, 2014

Dear Mr.BinayaJha

We are pleased to let you know that we are going to organize a blood donation program. It’s one of the annual activities of our
company. Many people die each year due to the lack of blood in their treatment. In this regard, I would like to present a case in
which we contributed to saving life through blood donation. A few years ago, an employee of Express Welfare was driving to a
friend’s wedding when an oncoming car, operated by a drunk driver, swerved across the center line. Brad doesn’t remember the
crash. But he does remember two months spent in the hospital, two months of surgery and therapy.

Without the help of people like us, Saurav would not have lived. Some Express Welfare employees save lives regularly. We’re blood
donors. Please be a lifesaver and join us on Friday, March 19th, for Express Welfare’s annual blood drive.

Your help is needed for a successful drive.

Giving blood is simple. The entire process will take less than 45 minutes.

Giving blood is safe. Experienced health professionals from the Red Cross Society will be on-site to conduct the procedure exactly
as they would in a clinic setting.

Giving blood is convenient. The Red Cross staff will be in Room 401, Building B, between 9:00 A.M. and 3:00 P.M. To save time,
make an appointment to donate. Call the Red Cross Blood Center at 569-1170.

Giving blood is important. Nobody knows who will need blood next, but one thing is certain—it will be available only if healthy,
caring people take time to give it. Saurav’s accident required 110 units—more than 12 gallons—of blood. Because 110 people set
aside 45 minutes, Saurav Nepal has a lifetime of minutes to be grateful.

Take a few moments now to make your pledge on the reverse side of this letter. Then return it to the Community Relations
Department, Mail Location 12, by March 15th. For more information about the drive, call the Red Cross Society at 552-7116.

From Saurav and from other families—like yours and mine—who might need it in the days to come,

Sincerely,

HemantaShrestha
Manager

3. Why are meeting minutes important? What should be done while minuting? (4+6=10)
Answer:
It is important to capture the essence of the meeting, including details such as:
- decisions made (motions made, votes, etc.)
- next steps planned
- identification and tracking of action items

Minutes are a tangible record of the meeting for its participants and a source of information for members who were unable to attend.
In some cases, meeting minutes can act as a reference point, for example:
- when a meeting’s outcomes impact other collaborative activities or projects within the organization
- minutes can serve to notify (or remind) individuals of tasks assigned to them and/or timelines

The Institute of Chartered Accountants of Nepal 48


Suggested Answers - Business Communication CAP-II Examination - December 2014

Tips for record taking


Generally, meeting minutes usually include the following:
- Date and time of the meeting
- Names of the meeting participants and those unable to attend (e.g., “regrets”)
- Acceptance or corrections/amendments to previous meeting minutes
- Decisions made about each agenda item, for example:
o Actions taken or agreed to be taken
o Next steps
o Voting outcomes
o Motions taken or rejected
o Items to be held over
o New business
o Next meeting date and time

Tips for minuting:


Create an outline – having an outline (or template) based on the agenda makes it easy for you to simply jot down notes, decisions,
etc. under each item as you go along. If you are taking notes by hand, consider including space below each item on your outline for
your hand-written notes, then print these out and use this to capture minutes.
Check-off attendees as they enter the room - if you know the meeting attendees, you can check them off as they arrive, if not have
folks introduce themselves at the start of the meeting or circulate an attendance list they can check-off themselves.
Record decisions or notes on action items in your outline as soon as they occur to be sure they are recorded accurately
Ask for clarification if necessary – e.g., if the group moves on without making a decision or an obvious conclusion, ask for
clarification of the decision and/or next steps involved.
Don’t try to capture it all – you can’t keep up if you try to write down the whole conversation, so be sure to simply and clearly
write or type just the decisions, assignments, action steps, etc.
Record it – literally, if you are concerned about being able to keep up with note taking, consider recording the meeting (e.g., on your
smart phone, iPad, recording device, etc.) but be sure to let participants know they are being recorded and it can come in handy if
you need clarification.

4. What would you include while writing a persuasive resume or CV? 10


Answer:
Writing a persuasive resume is well worth the effort. Regardless of the job market, it is important to have a persuasive and well
written curriculum vitae, or CV, also known as a resume. The focus of a CV is experience, skills and achievements. Employers hire
people who are qualified and demonstrate an understanding of their business.

The following points should be considered focusing on the important components to be included in a persuasive CV.
- Remove redundant information from your resume. Do not mention past job functions that do not highlight skills or point out
accomplishments relevant to the job you are applying to.
- Tailor the CV to the advertised position. Carefully review the advertisement and show how your skills and experiences fit the
job requirements.
- Use the same keywords and terminologies that appear in the job description. The resume should demonstrate that the applicant
understands the employer's business and industry. Some resumes are scanned by computers: having matching keywords may
stop your resume from getting filtered out before an actual person has had a chance to see it.
- Select a simple design for your CV. The length will depend on your education and work experience, but keep it brief. Hiring
managers may be reviewing dozens of resumes, so they will not have time to read through a CV that reads like an essay.
- Set yourself apart. Mention unique experiences or unusual skills that may not be related to the position. Mention that you speak
a foreign language and that you have traveled extensively. Talk about your community activities, such as coaching the school
football team or volunteering at the local food bank.
- Summarize your qualifications for the position advertised. Use a brief statement of your experience and skills as the headline for
your resume.
- Avoid applying for positions that require skills that you don't possess. Your resume will come across as unpersuasive. Even if
you get an interview, it may turn out to be a waste of valuable time.
- Keep in mind the main purpose of the resume. The resume is not intended to be a complete biography. Its main purpose is to get
you an interview. Remember that persuasion requires clarity and precision.

The Institute of Chartered Accountants of Nepal 49


Suggested Answers - Business Communication CAP-II Examination - December 2014

5. Briefly explain any FOUR of the following: (4×2.5=10)


a) Kinesics
b) Overcoming communication barriers
c) Interpersonal communication
d) Pros and cons of workforce diversity
e) Techniques to close the job interview
Answer:
a) Kinesics
Kinesics is one of the important types of nonverbal communication. When communication takes place with the help of the
movement of the body parts such as head, hands, face, etc., it is known as kinesics. The eyes, fingers, legs, body postures, etc.
also have very important role to convey very specific meaning in a very specific context. Kinesics can be more effective than
the linguistic tool such as words and utterances in some specific context of communication. For example we can communicate
by nodding our head, blinking eyes, waving hands, and so on. Such movements can express more effectively our strong
feelings, intentions, and affective conditions.

b) Overcoming communication barriers


Communication systems may face a number of challenges such as interruption, interference, lack of knowledge in the
participants, etc. These obstacles and challenges are called communication barriers. These are mainly of two types: physical and
mental. Physical barriers are related to noises and physical disturbances. Mental barriers are related to the lack of linguistic as
well as cultural knowledge, emotions, etc. communication barriers need to be overcome because they may result in
organizational failure. In order to overcome such communication barriers the following strategies may be useful.
• Using familiar words and terms: Start with the familiar words to introduce a new idea. As inception of a new idea causes
resistance, the safest way to discuss and accept any new information is to begin with what is known. Start from what it is to
what it could be.
• Understanding traits of cross-cultural communication traits
• Understanding perceptions and conceptions of others
• Using listener-oriented viewpoints
• Avoidance of ambiguous words, jargons, slang and semantic words

c) Interpersonal communication
Interpersonal communication is the sending and receiving of messages between two or more people. Everyone communicates in
life and business, however, various barriers can exist that distort the message. Individuals must be able to identify the barriers
and find ways to eliminate them in a way that improves the communication flow. Learning a few techniques or using proper
preparation can help individuals tailor their communication in a way that provides focus and improves the message.

Effective interpersonal communication is an ongoing process. As new messages come up or audiences change, senders must
tailor their messages to ensure effective communication. Failing to observe the barriers in communication can result in more
time clarifying the message than creating it. Senders will often spend more time correcting the message, which can result in
more time spent on a basic message.

d) Pros and cons of workforce diversity


Having a diverse workforce can benefit consumers, work teams, and business organizations. However, diversity can also cause
divisiveness among identity groups. Business communicators should be aware of and sensitive to differences in the
communication techniques of men and women. To promote harmony and communication in diverse workplaces, many
organizations develop diversity training programs. You must understand and accept the value of differences. Don’t expect
conformity, and create zero tolerance for bias and prejudice. Learn about your cultural self, make fewer assumptions, and seek
common ground when disagreements arise.

e) Techniques to close the job interview


Once the interview nears conclusion, start thinking about how to end on a positive note. It is easy to become flustered after a
challenging interview, so as an interviewee one need to be sure to practice questions that you plan to ask. Also, focus on how to
leave a lasting positive impression.
• Asking your own questions: Be prepared with meaningful, thoughtful questions to help you determine whether this job is
right for you.
• Ending the interview positively: Summarize your strongest qualifications, show your enthusiasm for the job, and thank the
interviewer. Ask for the interviewer’s business card. Shake hands, and acknowledge anyone else on the way out.

The Institute of Chartered Accountants of Nepal 50


Suggested Answers - Marketing CAP-II Examination - December 2014

Marketing

Section -'B'

1. Read the following case and answer the questions given below:
According to a report published in one of the popular daily newspapers of Nepal on September 13, 2011, Government lab test
confirmed contamination in Gudpak, a popular sweet produced by Corner TajaGudpakBhandar and Shree Krishna Gudpak at New
Road along with Kanhaiya Brand Ghee.
The Department of Commerce had sent samples of the Gudpak and Kanhaiya Ghee to the Department of Food Technology and
Quality for tests. The report found thatKanhaiya Ghee was substandard and contained 16 percent fat against the needed 28 percent. It
was not safe to consume. It posed a risk to people’s health,The food technology department sent the report to the Kathmandu District
Administration Office (DAO).

The DAO assured strict action as per the Black Marketing and Some Other Social Offences Punishment Act against those found
guilty of producing low quality food.As per the Consumer protection Act, the government can send the offenders to 14 years in jail
and fine them up to Rs. 500,000.

Meanwhile, a monitoring team from the Kathmandu DAO and the food technology department inspected Pushpa Dairy. The
inspection team found the dairy using a broken filter system.Contaminated water was being mixed in the milk. The inspection team
destroyed some 1,000 litres of contaminated milk and ordered the firm to change the filter system, label the products properly and
not to use powdered milk until they come up with a lab report.
Questions:

a) Which marketing concept was applied by the firm and why? 5


b) What should be the responsibility of business community in society in the context of the above case? 5

Answer According to the situation given in the case, the marketing firms should adopt the societal marketing concept because the
conduct of marketing firms affect the whole society and its welfare.
The punishment decision taken by the government was appropriate, but it should create awareness among the business
community regarding their role in social welfare and well-being.It should not be a window dressing; it should be a continuous
process.
Since the final users of any product are consumers, they must be aware of the consequences of the use of product. Therefore,
before making buying decision, they must have adequate information about the products and their impacts.
The business community must understand the business ethics and should be responsive to the society. Only those communities
can sustain their business in this competitive world which are aware of social obligation and work for social welfare and well-
being.
2.
a) What do you know about selling concept of marketing? Explain its features. 5
b) What is marketing environment? Identify the components and features of macro environment. (2+3=5)
Answer a)
Selling concept holds that consumers and businesses, if left alone, will ordinarily not buy enough of the products without selling and
promotion effort. Therefore organizations must undertake aggressive selling and promotion efforts. The selling concept is practiced most
aggressively with unsought products, products that buyers normally do not think buying, such as insurance policy and encyclopedias.
Most firms practice the selling concept when they have overcapacity. Their aim is to sell what they make rather than make what the
market wants. Such marketing carries high risks. It focuses on creating sales transactions rather than on building profitable relationships
with customers. It assumes that consumers who are coaxed into buying the product will like it. These are usually poor assumptions to
make about buyers. Most studies show that dissatisfied customers do not buy it again. Whereas the satisfied customer tells three others
about good experiences, the average dissatisfied customer tells ten others about bad experiences.

The features of selling concept are


1. It satisfies seller's needs.
2. It aims to sell what it made.
3. Aggressive selling and promotion
4. Profit through high sales volume.

The Institute of Chartered Accountants of Nepal 51


Suggested Answers - Marketing CAP-II Examination - December 2014

b) Marketing environment encompasses all those surroundings or forces which are constantly changing and carry with them both
opportunities and threats. Macro environmental forces include the following components –
i. Economic Environment
ii. Demographic Environment
iii. Socio-Cultural Environment
iv. Technological Environment
v. Political and Legal Environment
vi. Natural Environment
vii. Competitive Environment
These forces have distinct features, which are as follow:
• These forces are external in nature and cannot be controlled or manipulated by the marketer when required;
• Forces in marketing environment are dynamic in nature and go on changing over time, such as habit of people, their needs
and wants, technology, demographic character of the people, politics, rules and regulations, competition, etc.
• These forces create opportunities as well as threats to the management.Without their consideration smooth functioning of
management is not possible.
• Marketing environmental forces are potentially relevant to the marketing decision-making.without the consideration of
marketing environment marketing program can neither be formulated nor implemented successfully.
• Marketing environment is the source of not only threats to the management but also the source of opportunities. A
marketer identifies business opportunities from among the marketing environmental forces.
3.
a) Give the meaning of marketing research and also point out the marketing research process. (2+3=5)
b) Describe the meaning and importance of market segmentation. (2+3=5)
Answer a)
Marketing managers often commission formal marketing studies of specific problems and opportunities, such as a market survey, a
product-preference test, a sales forecast by region or an advertising evaluation. We define marketing research as the systematic design,
collection, analysis and reporting of data and findings that are relevant to a specific marketing situation facing the company.
A company can obtain marketing research in a number of ways. Most large companies have their own marketing research departments.
Some small companies can hire the services of a marketing research firm or conduct research in creative and affordable ways. They can
engage students or professors to design and carry out projects, they can use the Internet, and they can visit competitors.
Companies normally budget marketing research at 1 to 2 percent of company sales. Much of this budget is spent with outside research
firms, which fall into three categories. Syndicated-service research firms gather consumer and trade information, which they sell for a fee.
Custom marketing research firms design studies, carry them out, and report the findings. Specialty-line marketing research firms provide
specialized services such as field interview.
Effective marketing research process can be mentioned as follows:
1. Define the problem and research objectives
2. Develop the research plan
3. Collect the information
4. Analyze the information
5. Present the findings
b) Market segmentation is a process of dividing the total market into several smaller, internally homogeneous groups. It is based on
customer oriented philosophy. No marketer can satisfy the needs of all customers by developing single marketing mix. The essence
of segmentation is that the members of each group are similar with respect to the factors that influence demand. In segmentation, we
first identify the wants of customers within a submarket and then decide if it is practical to develop a marketing mix to satisfy those
wants.
According to Philip Kotler, “Market segmentation is the subdividing of a market into homogeneous subsets of customers, where any
subset may conceivable be selected as a market target to be reached with a distinct marketing mix.”
There are several important reasons why businesses should attempt to segment their markets carefully. They are summarized below:
i. Better matching of customer needs: Customer needs differ. Creating separate offers for each segment makes sense and
provides customers with a better solution.
ii. Enhanced profits for business: Customers have different disposable income. They are, therefore, different in how sensitive
they are to price. By segmenting markets, businesses can raise average prices and subsequently enhance profit.
iii. Better opportunities for growth: Market segmentation can build sales. For example, customers can be encouraged to "trade-
up" after being introduced to a particular product with an introductory, lower-priced product.

The Institute of Chartered Accountants of Nepal 52


Suggested Answers - Marketing CAP-II Examination - December 2014

iv. Retain more customers: Customer circumstances change, for example they grow older, form families, change jobs or get
promoted, change their buying patterns. By marketing products that appeal to customers at different stages of their life ("life-
cycle"), a business can retain customers who might otherwise switch to competing products and brands.
v. Target marketing communicationbusinesses need to deliver their marketing message to a relevant customer audience. If the
target market is too broad, there is a strong risk that the key customers are missed and the cost of communicating to customers
becomes too high. By segmenting markets, the target customer can be reached more often and at lower cost.
vi. Gain share of the market segment: Minor brands suffer from lack of scale economies in production and marketing,
pressures from distributors and limited space on the shelves. Through careful segmentation and targeting, businesses can often
achieve competitive production and marketing costs and become the preferred choice of customers and distributors. In other
words, segmentation offers the opportunity for smaller firms to compete with bigger ones.

4.
a) What is service product? Explain its distinct features. (2+3=5)
b) What is pricing? Explain the importance of pricing to economy. (2+3=5)
Answer a)
A service product may be defined as any task performed by the company as 'provision' of facility or product. Services may or may not
be attached with the physical product. If the main goal of the company is to provide services rather than a physical good, then services
become the main 'product' to the company. For example, repair and maintenance, medical and health care, banking, insurance,
research and consultancy, transportation, communication etc.
Characteristics of Services
Service product has distinct features as follows:
• Services are intangible (Intangibility): Services are abstract in nature, so they cannot see, feel, taste, hear, or smell. Before
they are purchased, customer cannot handle, examine or tried out.
• Services are perishable and cannot be stored (Perishability): Services disappear quickly; they are perishable in nature and
have no stored value. For example, if a dentist's customer fails to make an appointment, a half-hour or a certain time of the
dentist is gone forever.
• Services vary widely in quality (Variability in quality): Services are heterogeneous in nature. It is impossible for a service
industry, or even and individual seller of services, to standardize service output, this is because the skills of several providers of
services may vary widely, and even a single service provider may provide different quality services to the customers. For
example, an airline does not provide the same quality of service on each trip.
• Services are not separable from their provider (Non-seperability): Services are inseparable from the supplier or service
provider. In services marketing, the service delivery agents or service sellers become the surrogates or symbol of the quality of
service. For example, if the customers are crazy of services from a particular dentist because of his quality services. But in the
mean time, instead of inspection by himself, a substitute dentist is supplied, the customers will be dissatisfied as such customers
may divert to another dentist they feel superior or better. Hence, services are nontransferable, i.e., they cannot be transferred to
another party through the channels of distribution like tangible goods.
• Most of the services are purchased and consumed at production site. For example, to get dental services, customers have to
visit to the dentist's clinic: for the maintenance of bicycles, the bicyclist has to visit to the repair and maintenance workshop, etc.
• Non-measurability: Like physical items, services do not have physical features like shape, weight, colour, design, length,
breadth, etc. Therefore, service items cannot be measured like physical items.
b) Price is the exchange value of the product or service expressed in terms of money. It is the amount of money needed to buy the
product. In another words, price is the amount of money that customers pay for the product or service. It is what customers pay for
what they get.
Pricing refers to an act of determining the exchange value between what buyers get and what the sellers receives. It is a process of
setting prices for various products and services. In short, pricing is the act of determining exchange value of a product or service.
Pricing involves formulating pricing objectives, policies and strategies, setting base price, determining discount and commission etc.
Pricing is the most visible but secretly performed activity in a business organization.
We can explain the importance of pricing to economy as follows:
1. Price influences factors of production
Rent, wages, interest and profit are paid as price for land, labor, capital and entrepreneurship. Changes in rent, wages, interest
and profit influences the demand and supply of the factors of production.
2. Price determines demand and supply
From the economic point of view, high prices increase supply and low prices increase demand. The level of demand and
supply are determined by price. The price of the products determines the numbers of buyers whose purchasing power allows
them to buy it.

The Institute of Chartered Accountants of Nepal 53


Suggested Answers - Marketing CAP-II Examination - December 2014

3. Price affects saving and investment


High prices discourage saving and investment. Government controls prices to encourage saving and investment. They provide
subsidies to stabilize and lower prices. Inflation also affects price.
4. Price is a tool for economic management
Government use price in the form of taxes to manage the economy. They administer prices for public utilities like electricity,
water, telephone, postal services. They regulate prices of public enterprises.

5. Briefly explain the following: (5×2=10)


a) Benefits of niche marketing
b) Importance of warehouse in distribution system
c) Specialty product
d) Promotion
e) Reference groups
Answer a)
A niche is more narrowly defined group seeking a distinctive mix of benefits. Marketers usually identify niches by dividing a segment
into sub-segments. For example, the segment of heavy smokers includes two niches: those who are trying to stop smoking and those who
don’t care.
In an attractive niche, customers have a distinct set of needs; they will pay a premium price to the firm that best satisfies their needs: Its
benefits are that niche is not likely to attract other competitors. The nicher gains certain economies through specialization. The niche has
size, profit and growth potential.

b) Warehouse may be a place (open or closed) where goods are stored for certain period for the distribution of goods to the target
market whenever and wherever required. While warehousing is a specialized system and process of storing surplus goods (keeping
buffer stock) for use or consumption in future because goods may not have demand immediately after the goods are manufactured.
There are three types of warehouse –Private warehouse, Public warehouse and Bonded warehouse
Warehouses play an important role in the following ways –
Importance of Warehousing
Warehouses play an important role in the following ways –
i. Lengthens the life of products and maintains the product quality.
ii. Reduces the level of product damage and losses.
iii. They keep buffer stock of goods and facilitate supplying whenever demand creates in the market.
iv. Provides the facilities of marketing activities like; bulk-breaking, packaging, labeling, selling goods in small lots, etc.
v. Makes available the seasonal products in off-seasons also.
vi. Helps meeting required demand of the customers living in diverse geographical areas.
c) Specialty products are consumer products and services with unique characteristics or brand identification for which a significant
group of buyers is willing to make a special purchase effort. Buyers normally do not compare specialty products. They only invest
the time needed to reach dealers carrying the wanted products. Customers are usually willing to travel great distance to buy
specialty product. Dealers do not need convenient locations, although they must let prospective buyers know their locations. Bata
shoes, Mercedes car are examples of specialty product.
d) Promotion is one of the major elements of marketing mix which provides valuable information to consumers about product, price,
availability, utilities and benefits. Promotion consists of various activities that facilitate exchanges with target customers through
persuasive communication which stimulates the demand of the product.
According to Philip Kotler, “Promotion includes all the activities the company undertakes to communicate and promote its products
to the target market.”
Promotion includes various types of activities such as advertising, sales promotion, personal selling, publicity and public relation to
inform, persuade, remind and reinforce the target market about the company’s offerings. These activities influence the customer’s
feelings, belief or behavior for buying.
e) Reference groups have a direct/ face to face or indirect influence on a person’s attitudes or behavior. Groups having a direct
influence on a person are called membership groups. Some primary groups are family, friends, neighbors, and co-workers, with
whom individuals interact fairly continuously and informally. Secondary groups, such as professional and trade union groups tend
to be more formal and require less continuous interaction. Reference groups expose people to new behaviors and lifestyles,
influence attitudes and self-concept and create pressures for conformity that affect product and brand choices.

The Institute of Chartered Accountants of Nepal 54


Suggested Answers - Income Tax & VAT CAP-II Examination - December 2014

Income Tax & VAT


1.
a) Saru Pvt. Ltd., Kathmandu declared a Voluntary Retirement Scheme (VRS) for its staffs during FY 2071/72. Provident funds are
deposited in CIT, an approved retirement fund. Man Bahadur decided to take VRS with effect from 1st Baishakh 2072. The
following details are available for income of Man Bahadur for the year.
i) Basic Salary till Chaitra 2071 Rs. 50,000 per month.
ii) Monthly Allowance Rs. 30,000 per month.
iii) Provident Fund contributed by employer 10% of basic salary
An equal amount was contributed by employee.
iv) Life Insurance premium paid by Man Bahadur for himself Rs. 25,000
v) Remote area allowance for working in.
Category C district for two months Rs. 5,000 per month.
vi) Provident Fund paid by CIT on 15th Baishakh 2072 Rs. 12,00,000.
Retirement payment paid by Saru Pvt. Ltd. as per VRS was Rs. 20, 00,000, paid to Man Bahadur directly by the company.

Required:
(a) Calculate taxable income and tax liability of Man Bahadur for F/Y 2071/72 as per Income from Employment. Assume
individual. 7
(b) Identify any payments subject to final withholding Tax (Final TDS) and applicable TDS rate and Tax amount. 3

b) JKL Limited purchased 100 shares of MNO Bank Limited for each Rs. 200 each including brokerage and other expenses, as on
Chaitra 19, 2058, the company was holding 250 shares of MNO including the shares purchased and bonus shares paid by MNO.
The quoted closing price on Chaitra 18th, 2058 on MNO shares was Rs. 800 each.
Calculate the gain from the disposal of the shares under the circumstances given below: 10
i) On Jestha 15, 2071 the company had disposed of the total shares at the rate of Rs. 1,000 each. The brokerage paid was
1.2% of the sales amount.
ii) On Jestha 15, 2071 the company had disposed the total shares at the rate of Rs. 750 each. The brokerage paid was 1.2% of
the sales amount.
iii) On Falgun 10, 2070 the company had received 125 bonus shares for the 250 shares held by it. On Jestha 15, 2071 the
company had disposed 250 shares at the rate of Rs. 750 each. The brokerage paid was 1.2% of the sales amount.

Answers:
1a) Answer to part a) of question 1(a)
Calculation of taxable income and tax liability of Man Bahadur for FY 2071/72
Details Amount Notes
Salary 450,000.00
Monthly Allowance 270,000.00
Provident Fund by Employer 45,000.00
Remote Area Allowance 10,000.00
Total Assessable Income 775,000.00
Less: Contribution to Approved Retirement Fund (90,000.00) 1
1/3rd of Assessable Income 258,333.33
Contribution 90,000.00
Maximum 300,000.00
Taxable Income 685,000.00
Less: Life Insurance Premium (20,000.00) 2
Less: Remote Area Benefit (5,000.00) 3
Net Taxable Income 660,000.00
Calculation of tax liability for FY 2071/72
Tax Slab Tax Liability
1st Rs. 250000 @1% 2500
Next Rs. 100,000 @15% 15000
Balance Rs. 310,000 @25% 77500
Total Tax Liability 95,000

The Institute of Chartered Accountants of Nepal 55


Suggested Answers - Income Tax & VAT CAP-II Examination - December 2014

• Note 1- Contribution to approved retirement fund is deductible upto 1/3rd of assessable income or actual contribution or Rs. 3
lakh whichever is lower. Here, the amount deductible is Rs. 90,000.
• Note 2- Life Insurance premium paid is deductible actual premium paid not exceeding Rs. 20,000.
• Note 3- Remote area benefit available for Category C district is Rs. 30,000, proportionate for period of stay. Here, two
months.

Answer to part (b) of question 1(a)

The retirement payments made to Man Bahadur are subject to TDS as per Sec. 88 of the Act.
a. Provident Fund paid by CIT
Total Payment (Note 1) Rs. 12,00,000
Less 50% of the payment or Rs. 5,00,000 whichever is higher (Rs. 6,00,000)
Net Payment subject to tax Rs. 6,00,000
TDS @ 5% Rs. 30,000
The payment is final TDS as per Sec. 92.
Note 1: The payment for provident fund is assumed to have related only with the period after the Income Tax Act, 2058.

b. Retirement Payment paid by employer is subject to 15% tax as per Sec. 88. The tax amount for Man Bahadur is Rs. 3,00,000.
The payment is final as per Sec. 92 of the Act.
1b)
Solution under the first circumstance:
Cost of shares (100 shares @ Rs. 200 each Rs. 20,000
Cost of bonus shares (150 Bonus Share) Nil
Cost of 250 shares Rs. 20,000
But the index value of the shares as on Chaitra 18, 2058 was Rs. 800 per share and so the deemed cost of the 250 shares shall be
Rs. 800×250=Rs. 200,000 + Rs. 3,000 (1.2% of the sale proceeds for brokerage on sales.)
The amount received from the disposal is 250×Rs. 1000=Rs. 250,000.
Thus the gain is of Rs. 250,000-Rs.203,000=Rs. 47,000.

ii. Solution under the second circumstance:


The index value of the shares as on Chaitra 18, 2058 was Rs. 800 per share and so the deemed cost of the 250 shares shall be Rs.
800×250=Rs. 200,000 + Rs. 2,250 (1.2% of the sale proceeds for brokerage on sales.)
The amount received from the disposal is 250×Rs. 750=Rs. 187,500.
Thus the loss is of Rs. 187,500-Rs.202,250=Rs. 14,750.

iii. Solution under the third circumstance:


The index value of the shares as on Chaitra 18, 2058 was Rs. 800 per share and so the deemed cost of the 250 shares shall be Rs.
(800×250) = Rs. 200,000
Plus the cost of 125 bonus shares received after Chaitra 18, 2058 = Rs. Nil
The total cost of 375 shares is Rs. 200,000 and so the cost of one share comes to Rs. = 533.33.
The cost of 250 shares sold comes to Rs. 133,332.50+2,250 (1.2% of the sale proceeds for brokerage on sales).
The amount received from the disposal is 250×Rs. 750=Rs. 187,500.
Thus the gain is of Rs. 187,500-Rs. 135,582.50 =Rs. 51,917.50.

2.
a) Mr. Rich has purchased a private building amounting to Rs. 2 crores on Kartik 23, 2060 at Kathmandu. He sold such building
on Rs. 3 crores on Poush 26, 2070. During such period, Mr. Rich was gone abroad, intermittently for a period of 120 days. Will
such building be considered as "Non-Business Chargeable Assets"? What will be your answer if such building was sold after
two months (i.e. on Falgun 26, 2070)? 5

b) Munal Trade Link is a various goods supplier. The trade link is using accrual basis of accounting for the income generated
from the business. He gave an application to the department to change his basis of accounting from accrual to cash in Income
Year 2070/071. In accordance with the generally accepted accounting principles, Income Tax Department has given the
approval to account for his income on cash basis in Income Year 2070/71. Some accounting information of the trade link at the
end of 2069/070 before changing the basis of accounting, was as below;
i) Goods worth of Rs. 75,000 has already been supplied to a retailer in Income Year 2069/070 but amount has not been
received. Advance Rs. 45,000 has been received from another retailer.
ii) Goods worth of Rs. 60,000 has been purchased from a vendor in Income Year 2069/070 but amount has not been paid.
Advance Rs. 35,000 has been paid to another vendor.

The Institute of Chartered Accountants of Nepal 56


Suggested Answers - Income Tax & VAT CAP-II Examination - December 2014

iii) Communications and electricity expenses of Rs. 3,500 were paid on Bhadra, 2071. These expenses were incurred in Jestha
and Ashadh, 2070.
State how you make adjustments of income and expense in FY 2070/071 in above conditions? 5

c) Mr. Komal purchased a piece of land at Rs. 30 lakhs. He sold the land at Rs. 45 lakhs. He paid registration expenses Rs. 2
lakhs for this land. In this case, what would be the tax implications on the following situations?
i) The land was purchased on Chitra 2064 and sold it on Magh 2070.
ii) The land was purchased on Magh 2067 and sold it on Magh 2070.
iii) The land was purchased on Chitra 2059 and sold it on Baishak 2071.
iv) If selling and buying of the land were completed through a sole shareholder of a Pvt. Ltd. The shareholder is Mr. Komal.
5
d) Standard Chartered Bank has appointed AX Consultancy Pvt. Ltd. as Tax Consultant on Magh 20, 2070 and the effective date
of contract is Shrawan 1, 2071. Under the contract, the bank is liable to pay an annual fee of Rs. 500,000. As per the contract,
an advance amount of Rs. 100,000 is to be paid on appointment date (i. e. on Magh 20, 2070). AX Consultancy Pvt. Ltd. is in
confusion whether the amount of Rs. 100,000 to be included in its income of Income Year 207/071 as the appointment letter is
already received during the Income Year. Advise. 5

Answers:
2a)
Section 2 (da) (2) of Income Tax Act, 2058 has excluded the following assets from the definition of "Non-Business Chargeable
Assets" in case of an individual;
A private building of an individual that has been;
o owned continuously for ten years or more; and
o lived in continuously or intermittently for a total period of ten years or more.

In the given case, Mr. Rich has owned his private building for a period of ten years in which first condition is satisfied. But he lived
in for a period of less than ten years in which second condition is not satisfied.
Hence, such building is to be considered as "Non-Business Chargeable Asset" as per Income Tax Act, 2058.
But, in case where Mr. Rich sold such building after two months (on Falgun, 26, 2070) second condition of 'lived in continuously
or intermittently for a total period of ten years or more' will also be fulfilled and hence such building shall not be considered as
"Non-Business Chargeable Asset".

2b)
As mentioned in Chapter 6 of Income Tax Act, 2058, if any person takes the approval from Inland Revenue Department for the
change in basis of accounting for the purpose of taxation, or his basis of accounting is changed because of various provisions
mentioned in the same Chapter, inclusion of his income, claim of expenditure shall be computed in such a manner that there is no
repetition or short of income or claim of expenditure in that Fiscal Year.

Accordingly, the following adjustments shall be made in Income Year 2070/71;


i) When goods are supplied, the income shall be recognized in accrual basis of accounting. So, no need to adjustment for the
goods of Rs. 75,000.00 that was included in Income Year 2069/70. Advance amounting Rs. 45,000.00 was not assessable
income for the Income year 2069/70 as per accrual basis of accounting, so, it shall be included in Income Year 2070/71 on
cash basis
ii) When goods are purchased, the expenses shall be recognized in accrual basis of accounting. So, no need to adjustment for the
goods of Rs. 60,000.00 that was deducted in Income Year 2069/70. Advance payment Rs. 35,000.00 was not deductible
expenses for the Income year 2069/70 as per accrual basis of accounting, so, it shall be deducted in Income Year 2070/71 on
cash basis.
iii) Communications and electricity expenses of Rs. 3,500.00 were also eligible expenses for the Income Year 2069/70 on
accrual basis of accounting, so no need to adjustment in Income Year 2070/71. After making adjustments of inclusion of
income and claim of expenditure in Income Year 2070/71, the income or expenditure shall be accounted on cash basis for
subsequent years.
2c)

Land and building disposed for a value less than 30 lakhs by any natural person is not treated non-business taxable assets as per
Section 2(Da). The disposal value more than the limit shall be taken into account in computing the gain and loss on income from
investment. Tax liability is calculated on the basis of the net gain from the disposal of those assets. Withholding tax is managed for
such transactions under section 95 Ka (3) of the Act for natural person. If the land is sold after holding it for more than 5 years the
tax is 2.5 % and if it is sold after holding it for less than 5 years the tax rate is 5%. The Land Revenue Office withholds the tax on
such net gain.

The Institute of Chartered Accountants of Nepal 57


Suggested Answers - Income Tax & VAT CAP-II Examination - December 2014

Calculation of net gain on this land:


Particular Amount Amount
Incoming Rs. 45 Lakhs
Outgoings Rs. 32 Lakhs
Purchase cost Rs. 30 Lakhs
Transfer expenses Rs. 2 Lakhs
Net gain Rs. 13 Lakhs

So, TDS shall be as follows in this case. Any exemption limit available reduces the tax liability of the natural person. This depends
on other taxable income of the tax payer and a tax return should be submitted for this.

i) The land was purchased on Chitra 2064 and sold it on Magh 2070. Mr. Komal sold the land after holding 5 years. The TDS
is Rs. 32,500.00 (2.5% of 13 Lakhs)
ii) The land was purchased on Magh 2067 and sold it on Magh 2070. He sold the land within 5 years. The TDS is Rs.
65,000.00 (5% of 13 Lakhs
iii) The land was purchased on Chitra 2059 and sold it on Baishak 2071. Mr. Komal sold the land after holding more than 10
years. However, the TDS is Rs. 32,500.00
iv) If selling and buying of the land were completed through a sole shareholder of a Pvt Ltd, it will be regarded as disposal of
business assets. There will be no TDS, but the net gain is calculated under Sec. 36 of the Act and net gain after set off with
any unrelieved lossis included as income from business under Section 7.

2d)
As per section 22(3) of Income Tax Act, 2058 a company has to account for its income on accrual basis for tax purpose. Further,
as per section 27(2), the time the payment is derived, incurred, made, received or otherwise taken into account for tax purpose
shall be treated as the time of quantification of amount. In the given case, AX Consultancy Pvt Ltd is a company and has to
account for its income on accrual basis for tax purpose. Performance of service is the time to recognize the income. The time the
payment is derived shall be treated as the time of quantification of the amount. Hence, advance payment is irrelevant to recognize
the income and shall be simply recorded as advance payment received in Income Year 2070.71. Since the effective date of
contract is Shrawan 01, 2071 the service will accrue only on Income Year 2071.72 and shall be recognized the income
accordingly.

3.
a) Super company is a proprietorship industry producing tobaccos. The Industry has total sales of Rs. 2,00,00,000 and taxable
income of Rs. 35,00,000 in Income Year 2070/071. The company has not submitted estimated tax return during the year. In
addition, it has not paid any taxes. You are requested to assess the total Tax liability for this industry upto Mansir 20, 2071.
5
b) Mr. Ramesh has been retired from Government of Nepal on 15th Jestha, 2071. He has received the following retirement
payments in Ashadh, 2071. He has not any other sources of income except salary. Remuneration tax already has been deducted
and deposited. Assume, no retirement payment was accrued at the commencement of this act in connection with this
employment.

Particulars Amount (Rs.)


Payment from GON against accumulated leave and medical allowances Rs. 6,00,000
Payment from Employee Provident Fund against contribution Rs. 7,00,000
Payment from Citizen Investment Trust against contribution Rs. 5,00,000
5

The Institute of Chartered Accountants of Nepal 58


Suggested Answers - Income Tax & VAT CAP-II Examination - December 2014

Answers:
3a)
The company is a proprietorship firm, so the tax rate is applicable as per schedule 1(1). Thirty percent tax rate for tobacco business
is applicable only for entity.

Assumption:
The taxpayer has selected for single.
Calculation of total tax liability for F/Y 2070/71
Total taxable income Rs. 35,00,000.00
Particulars Tax rate Tax amount Rs.
Upto Rs. 200,000.00 0% -
Next Rs. 100,000.00 15% 15,000.00
Balance Rs. 32,00,000.00 25% 8,00,000.00
Additional Tax on the tax amount levied @ 25% for taxable
income above 25,00,000.00 (10,00,000.00*25%) i.e on Rs.
2,50,000.00 40% 100,000.00
Total tax amount 9,15,000.00
Fees under section 117 (1)(ka) 2,000.00
Fees under section 117 (1)(ga) (WN 2) 3,333.33
Interest under section 118 (1) (WN 2) 64835.62
Interest under section 119 (from Kartik to Mansir 2071) @ 15 % of
Rs.9,15,000.00 22,875.00
Total tax liability till Mangsir end 2071 1008043.95

Working Note 2: Calculation of Fees and interest


• Under Section 117 (1)(ga): Total assessable income as per the section is Rs. 2,00,00,000.00, fees 0.1 % annually of Rs.
2,00,00,000.00 for 2 months (from Kartik to Mansir, 2071) is Rs. 3,333.33 or Rs. 100 per month whichever is higher.
• Under Section 118: total tax Rs. 9,15,000.00

Cumulative 90% of Actual Interest


Interest @
Tax Cumulative Paid Chargeable Period
Installments 15 % amount
Amount Tax Instalme Amount (d) (Months)
Rs.
Rs. (a) Amount (b) nt (c) = (b)-(c)
Poush end 3,66,000.00 3,29,400.00 0 3,29,400.00 3 12,337.5
Chaitra end 6,40,500.00 5,76,450.00 0 5,76,450.00 3 21,616.87
Ashad end 9,15,000.00 8,23,500.00 0 8,23,500.00 3 30,881.25
Total 64,835.62

If students assume a Couple u/s 50 for calculation of tax liability, and calculate the interest u/s 118 taking the difference
from 100%, full marks will be awarded.

3b)
i) Fifty percent of the paid amount Or Rs. 5 Lakhs, whichever is higher, shall be deducted on the lump sum retirement payment
from any Approved Retirement Fund or Nepal government in computing tax liability of any natural person as per section 65(1).

Further, section 88 (1) (1) states that in the case of the retirement payment made by the Government of Nepal or by the
approved retirement fund, at the Rate of Five percent shall be deducted from the benefits calculated pursuant to section 65(1).
Such retirement payment is final withholding as per section 92 (1) (Chha).

Retirement payment to Mr. Ramesh from retirement fund and Nepal government and 50 percent thereof are as follows:
Particulars Amount
Payment from GoN against accumulated leave and medical allowances Rs. 6,00,000.00
Payment from Employee Provident Fund against contribution Rs. 7,00,000.00
Payment from Citizen Investment Trust against contribution Rs. 5,00,000.00
Total amount (WN 1) Rs. 18,00,000.00
50% of Total Payment (a) Rs. 9,00,000.00
Fixed amount u/s 65 (b) Rs. 5,00,000.00
Deduction allowed for the higher amount (a) or (b) above Rs. 9,00,000.00
Taxable Amount RS. 9,00,000.00

The Institute of Chartered Accountants of Nepal 59


Suggested Answers - Income Tax & VAT CAP-II Examination - December 2014

The EPF will withhold Rs.10,000.00 @ 5% on Rs. 2,00,000.00 (Rs.7,00,000.00-Rs.5,00,000.00) and similarly, the CIT will not
withhold any amount being only Rs.5,00,000.00. But, the GON will withhold Rs. 35,000.00 (i.e Rs. 45,000.00 @ 5% on
9,00,0000.00 minus the amount Rs. 10,000.00 already withheld by the EPF). It will be a final withholding u/s 92.

4. Write short notes of the following with reference to Income Tax Act, 2058. (5×2=10)
a) Advance ruling and public circulars are different in nature.
b) Not all persons are required to submit Income Tax returns.
c) Entity
d) Tax exempted incomes
e) Right of Tax payer.

Answers:
4a)
a. Advance Ruling and Public Circulars are different in nature.
Public Circulars are issued to general public to achieve consistency in the implementation of Act and to assist the tax officers
and tax payers in application of the provisions of the law. The public circular is mandatory for the department, but not for the
public. If the tax payer is not satisfied with the circular issued by IRD, he may choose for legal recourse. Detailed provisions
are stated in Section 75 of Income Tax Act.

Advance Ruling is issued to the applicant tax payer for clarifying application of tax law on proposed arrangement of the
applicant. The advance ruling is only applicable for the concerned applicant and is not available for general public. Detailed
provisions are stated in Section 76 of Income Tax Act.
4b)
b. Not all persons are required to submit income tax returns.
There are certain exceptions to provisions requiring submission of tax returns by the tax payer. As per Sec. 97, the following
persons are not required to submit tax returns:
• Person who do not involve in any economic activities leading to taxable income
• A natural person with only income from an employment based on a source in Nepal, with all employers during the
income year as resident persons and only one employer at a time and who only claims medical costs and retirement
contribution paid by the employer and do not claim donations or gifts paid to tax exempt organization.
• Person earning final withholding paymentsNatural persons owning public vehicles who pay presumptive tax as per
Schedule 1 of the Act.

4c)
Entity
As per section 2 (Bha) of the Act, entity is defined the following organization or body:
(1) A partnership, trust or company,
(2) Village Development Committee, Municipality or District Development Committee,
(3) Government of Nepal,
(4) Any foreign government or provincial or local government under that government or a public international organization
established by any treaty, or
(5) A permanent establishment of the organization or body referred to in clauses (1), (2), (3) and (4), which is not situated in a
country of which it is a resident.

4d)
As per Section 10 of Income Tax Act, 2058 the following amounts are tax exempted incomes:
i) Amounts derived by a person entitled to privileges under a bilateral or a multilateral treaty concluded between Nepal
Government and a foreign country or an international organization.
ii) Amounts derived by an individual from employment in the public service of the government of a foreign country.
Provided that-
a) The individual is a resident person solely by reason of performing the employment or is a non-resident person; and
b) The amounts are payable from the public funds of the country.
iii) Amounts derived from public fund of the foreign country by an individual who is not a citizen of Nepal as referred to in
paragraph (ii) or by a member of the immediate family of the individual.
iv) Amounts derived by an individual who is not a citizen of Nepal from employment by Nepal Government on terms of tax
exemption.
v) Allowances paid by Nepal Government to widows, elder citizens, or physically disabled individuals.

The Institute of Chartered Accountants of Nepal 60


Suggested Answers - Income Tax & VAT CAP-II Examination - December 2014

vi) Amounts derived by an exempt organization by way of-


a) Gift; or
b) Other contributions that are directly related to the organization’s function referred to in paragraph(s) (1) of the
definition of the exempt organization in section 2, whether or not the contribution is made in return for
consideration provided by the organization, or
c) Income earned by Nepal Rastra Bank as per its objective, or
d) Income earned by Securities Board of Nepal as per its objective.
vii) Pension received by a Nepali citizen retired from the army or police service of a foreign country provided the amounts
are payable from the public fund of that country.
viii) Any income of Nepal Government.

4 e)
Taxpayer’s Rights as per section 74 of Income Tax Act, 2058;
i. Tax Payer shall abide by the duties in accordance with this Act.
ii. A taxpayer with respect of paying tax under this Act shall have the following rights:-
a. Right to get respectful behavior;
b. Right to receive tax related information as per the prevailing laws;
c. Right to get opportunity of submitting proof in own favor in respect of tax matters;
d. Right to appoint lawyers or auditors for defense; and
e. Right to secrecy in respect of tax matters and keep it inviolable
[Clarification:- For the purpose of this section, taxpayer means a person whom the tax is imposed on and realized from as
referred to in section 3.]

5.
a) Explain whether the following statements are true or false with reasons: (5×1=5)
i) A person shall apply for the close of taxable transaction in the format prescribed in Schedule 10 of VAT Regulation, 2053.
ii) A person shall be imposed fine of Rs. 10,000 at each time in case of refusal for the inspection of books of account by the
Tax officer.
iii) The Tax paid by a diplomatic body or diplomat on the purchase of taxable goods or services shall not be refunded if the
amount purchase is less than Rs. 5,000 at one time.
iv) Tax Officer may assess Tax as per section 20 if Kathmandu District Development Committee collects the tax on taxable
transactions and deposit in its internal revenue account.
v) A foreign tourist can take refund of VAT on the purchase of goods amounting to Rs. 15,000 or more.

b) Apar International Pvt. Ltd. is a Recondition House involved in Purchase/Sale of Used Vehicle, Motorcycle, TV and
Refrigerator. The company purchases the used/old Vehicle, Motorcycle, TV and Refrigerator and Sale those goods to the
consumer by repairing and improving the same. Company purchased from Mr. Ram residing in Satungal at Rs. 20 lakhs and
after repairing the same sold to Ms. Sunita at Rs. 25 lakhs Apar Ltd. seeks your advise as how much VAT shall he collect on
such sales amount.
5
c) Saurav Publication prints the various books and sells to the wholesaler. The Company itself has constructed a building for its
Publication House. The cost of construction was as follows:
Labour charges Rs. 30,00,000
All Material (Cement, rod etc) purchase cost Rs. 5,50,000 inclusive VAT.

State the VAT Implications on this case, if applicable with references to VAT Act, 2052 and rules, 2053. 5

d) Ilam Tea Pvt. Ltd. is a tea processing and producing industry, located in Ilam. It sells the tea within Nepal. It has following
transaction in the month of Bhadra, 2071.
Particulars Transactions Amount (Rs.)
Sales:-
VAT registered party 20,00,000
VAT Non-registered party 5,00,000
Purchase:-
Wages and others that are VAT exempt 12,00,000

Compute the VAT amount payable/receivable by the Tea Industry with reference to the VAT Act, 2052 for the Month. Assume
no VAT on opening. 5

The Institute of Chartered Accountants of Nepal 61


Suggested Answers - Income Tax & VAT CAP-II Examination - December 2014

Answers:
5a)
i) False. Application for the close of VAT shall be filed in the format prescribed in Schedule – 11 of VAT Regulation, 2053.
ii) False. A person shall be imposed fine of Rs. 20,000.00 at each time in case of refusal for the inspection of books of account by
the tax officer.
iii) True. The tax paid by a diplomatic body or diplomat on the purchase of taxable goods or services shall not be refunded if the
amount purchase is less than Rs. 5,000.00 at one time.
iv) True. Tax officer may assess tax as per Section 20 if Kathmandu District Development Committee collects the tax on taxable
transactions and deposit in its internal revenue account.
v) False. A foreign tourist can take refund of VAT on the purchase of goods amounting to Rs. 25,000 or more.

5b)
a) Section 17(5) of the VAT Act, 2052 read with Rule 33 of VAT Rules 2053 provides that, in case of person dealing with used
assets, VAT shall be determined on the difference amount between sale value and purchase value only.
In the given case, on the basis of above provision, VAT shall be levied on Rs. 5 lacs only and accordingly VAT amount will be
Rs. 65,000.(being 13% of Rs. 500,000).

5c)

Saurav Publication is engaged in VAT exempt transaction as per Group 7 of Schedule 1 of the VAT Act, 2052.
If it has constructed a building for less than Rs. 50 lakh, it is not attracted u/s 8(3) which states that even though the construction of
a building or apartment or shopping complex and similar other structure as specified by the Department, of which value is more
than Rs. 50 lakhs, and which is built for business purpose is procured from a person who is not registered, tax shall be assessed and
collected from a person who has ownership in that structure as if such construction were procured from a registered person.

Rule 6 (Kha) is also not attracted as it mentions that any person who is constructing building, apartment, shopping complex or
similar structure as specified by the Department for business purpose amounting more than Rs. 50 lakhs, he has to get it
constructed only from VAT registered person.

Thus, VAT paid on purchase of the materials are not eligible for credit u/s 17 and will be capitalized in the cost of the building.

5d)
Schedule 1 of VAT Act, 2052 has provided refund facility for Tea Industry. As per the provisions, 50 % of collected VAT amount
on the sales to VAT registrants shall be refunded as specified by the Inland Revenue Department to the tea producing and
processing industry of Nepal.

The Ilam Tea Industry has collected VAT Amount in the Month of Bhadra, 2071 as follows:

Particulars Transactions Amount Rs. VAT collected Rs.


Sales
VAT registered party 20,00,000.00 2,60,000.00
VAT Non-registered party 5,00,000.00 65,000.00
VAT Payable 3,25,000.00

The industry should pay VAT amount of Rs. 3,25,000.00 on Aswin, 2071 for the month of Bhadra, 2071. It can apply for refund
amounting of Rs. 1,30,000.00 (50 % of Rs. 2,60,000.00). If the Inland Revenue Department permits the Tea Industry for the
adjustment of refundable amount on the payable amount, it shall pay the VAT amount accordingly. So, at the end of Bhadra, 2071,
VAT payable amount is Rs. 3,25,000.00 and VAT receivable amount is Rs. 1,30,000.00

6.
a) Finance Manager of ABC Hardware is worrying about the price adjustment with it's supplier. He had heard that he can do the
price adjustment through the use of Debit/Credit Note. You are requested to advise him regarding the content of Debit/Credit
Note as per VAT Rules. 5
b) Input Tax Credit (ITC) under sec. 17 of Value Added Tax (VAT) Act, 2052. 5
c) M/s RK Trading Private Limited is a Value Added Tax Registered company. It is engaged in the business of importing wines
& liquor and selling in local market. The company is in confusion to take tax credit on procurement of wines and liquor as per
Value Added Tax Act/Rules, Advise. What will be your answer if the company does not have such primary business? 5
d) Discuss the amount to be included in Taxable Value as per section 12 of Value Added Tax Act, 2052. 5

The Institute of Chartered Accountants of Nepal 62


Suggested Answers - Income Tax & VAT CAP-II Examination - December 2014

Answers:
6a)
As per Rule 20(1) of the Vat Rules, 2053 a person registered under VAT can adjust the price difference on Sale of goods or services
by issuing. Debit Note or Credit Note. As per the said Rule, the following are the content of Debit/Credit Note:
a. Serial Number
b. Date of Issue
c. Name, Address and PAN Number of Supplier
d. Name, Address and PAN Number (if he is registered) of Receiver
e. Invoice No. and Date of Concerned Transaction
f. Detail of Product or Service and Reason of Debit or Credit
g. Amount of Debit or Credit
h. Vat Debit or Credit amount.

6b)
The following provisions are mentioned for tax offset in section 17 of the VAT Act, 2052:

(1) A registered person can offset the amount of tax he has collected against the tax he had paid or due in importing or receiving
goods or services related to his own taxable transactions.
(2) Notwithstanding anything contained in Subsection (1), it may be provided that no deduction or only a partial deduction may be
granted in the case of the prescribed goods that can be used for personal purpose or for business purpose or for both purposes.
(3) If the entire portions of goods or services transacted in a month were not used for taxable transactions the tax previously paid on
the goods or services shall be offset as prescribed for the portion that was solely used for taxable transaction of the goods or
services.
(4) If goods or services for which offset privileges pursuant to this section have been allowed cease to be used for taxable taxation
before the end of its useful life, as may be prescribed, such goods or services shall be treated as sold at the immediate market
value and tax shall be collected.
(5) Notwithstanding anything contained in sub-section (1), offset privileges to be provided for a registered person who deals with
the used goods shall be as prescribed.
(5a) The concerned taxpayer shall be allowed to deduct the tax paid on the capital goods that were imported or purchased by entering
into a loan agreement under financial lease subject to Sub-section (2).
(5b) The concerned taxpayer shall be allowed to deduct tax paid pursuant to Sub-section (2) of Section 8, Section 12A and Sub-
section (3) of Section 15.
(6) The offset privileges under this Act shall be provided only when a claim is substantiated by documents as prescribed.
(7) The provision of offset on tax paid or payable on goods, which has remained unused at the time of the registration and is for use
in making taxable transactions, shall be as prescribed.
(8) Notwithstanding anything contained elsewhere in this Section, the name of a taxpayer who does not submit the tax returns for
Six months consecutively shall be made public, and if there is any amount due for tax deduction by such a taxpayer, such
amount may be suspended and his or her registration may also be suspended…

6c)
Rule 41(1) of Value Added Tax Rules, 2053 prescribes the following provision.

For the purpose of section 17 of the act, tax credit shall not be allowed in respect of the following goods or services;
- Alcohol or alcohol mixed beverages such as liquor, beer.
Rule 41(3) prescribes the following provision:
For a registered person, who carries on a business of those goods mentioned in sub- rule (1) and (2) of Rule 41 as the principal
business, there shall be no restriction for the tax credit in accordance with the procedures mentioned in these rules.

The principal business of M/s RK Trading Private Limited is of importing wines & liquor and selling in local market and tax credit is
allowed. If the company's primary business is not of importing wines and liquor and selling in local market, input tax credit shall not
be available as prescribed under rule 41(1).

6d)
Section 12 of Value Added Tax Act, 2052 of the act lists the amount to be included in Taxable value.

Section 12(1)
Except otherwise provided in this act, where only cash is the consideration, the taxable value shall be the price the supplier receives
from the recipient of supply.

The Institute of Chartered Accountants of Nepal 63


Suggested Answers - Income Tax & VAT CAP-II Examination - December 2014

Section 12(2)
The following amounts shall be included in the taxable value:
(a) The amount of all expenditures related to transportation and distribution, which was borne by a supplier in connection with the
transaction, and the amount of profit.
(b) Amount of excise duties, ownership fee and all other tax amount other than a tax imposed under is act.

Clarification – For the purpose of this section, “other tax amount” means duties, charges and fees prescribed by the annual Finance
Act.

Section 12(3)
Taxable value shall not include the amount of discount, commission or other similar commercial rebates granted in the value of
supplying goods or service.

Section 12(4)
The taxable value of any goods or service exchanged or bartered shall be equal to the market value of the goods or services so
exchanged or bartered.

Section 12(5)
Except otherwise provided in this act, while determining the taxable value of any imported goods all amounts of transportation,
insurance, freight, commission of agents and other persons, customs duties, countervailing duties plus any taxes if levied on imports
other than value added taxes, shall be added.

Section 12(6)
Where the value of any goods or services is found to be lower than the prevailing market value, the taxable value of such goods or
services shall be equal to the market value.

Section 12(7)
The taxable value of goods or services supplied for partial consideration shall be equal to the market value.

Section 12(8)
A deposit, given in respect of goods or services, shall not be treated as taxable value unless the supplier adjusts the deposit as
consideration for the supply as prescribed.

The Institute of Chartered Accountants of Nepal 64

You might also like